Sunteți pe pagina 1din 24

@

u
I
a

N
e
p
t
u
n
o
Centro de Estudios Aula Neptuno - GRANADA Ver.: 9/2001
www.aulaneptuno.com
- 1 -
1. Una mujer de 27 aos, primigravida, en la semana
35 de gestacion ha tenido una perdida subita de li-
quido amniotico 16 horas de entrar en la sala de
partos. No presenta contracciones. La exploracion
mediante especulo esteril revela una pequea can-
tidad de liquido en la vagina, que es positivo para
la cristalizacion en helecho. Se observa una dilata-
cion de 3 cm. y un borramiento del 80% en el cue-
llo uterino. Se toman cultivos vaginal y cervical pa-
ra Neisseria gonorrhoeae y para estreptococo beta-
hemolitico del grupo B. La monitorizacion fetal
revela un patron reactivo sin desaceleraciones. La
ecografia muestra un feto en presentacion de verti-
ce con un diamentro biparietal y medidas de longi-
tud de femur correspondientes a 35 semanas de
gestacion. El volumen de liquido residual es ade-
cuado. Cuales de los siguientes enfoques no es
apropiado para el tratamiento?

1. Esperar el inicio del parto
2. Inducir el parto
3. Obtener liquido amniotico para realizar estudio
de madurez fetal
4. Realizar un parto mediante cesarea
5. Administrar antibioticos profilacticos frente al es-
treptococo del grupo B mientras se obtienen los
resultados de los cultivos

2. Al preparar las directrices para el control prenatal
rutinario usted valora el screening para alfa-
fetoproteina en suero materno (AFPSM). Cuales
de las siguientes afirmaciones es correcta e incor-
poraria usted a las directrices?

1. El screening para AFPSM es necesario solo si hay
antecedentes familiares de defecto del tubo neural
2. Debe realizarse el screening para AFPSM durante
el primer trimestre
3. Una valoracion detallada y conjunta con la pacien-
te sobre el screening para AFPSM es necesaria so-
lo si se produce un resultado anomalo
4. Si el screening para AFPSM es normal, es necesa-
rio repetirlo 2 semanas despues
5. Es importante indicar en la ficha de la paciente que
se le ofrecio la posibilidad de realizar screening pa-
ra AFPSM, e incluir la respuesta de la paciente

3. Paciente de 34 a. Embarazada de 36 semanas
acude a urgencias tras presentar dolor intenso y
continuo en hipogastrio, acompanado de mancha-
do genital de sangre obscura de escasa cuantia.
Como antecedentes previos destaca hta leve que
ha controlado sin medicacion.Exploracion fisica:
palidez cutaneomucosa. Taquipnea moderada, 120
lpm. Acr sin hallazgos. Utero de tamano normal
para la edad gestacional, aumento de tono que di-
ficulta palpacion fetal. Se realiza ultrasonografia
que muestra tonos cardiacos a 170 lpm. Cual de
las siguientes diagnosticos de sospecha le parece el
mas probable:

1. Placenta previa.
2. Desprendimiento prematuro de placenta.
3. Parto en curso.
4. Sidrome HELLP.
5. Rotura prematura de membranas.

4. En la amniocentesis precoz para el diagnstico
prenatal es FALSO:

1. Se realiza por puncin transabdominal.
2. El momento idneo es entre la 17 y 19 semana de
gestacin.
3. Util para el estudio del cariotipo fetal.
4. Sirve para el diagnstico de enfermedades meta-
blicas.
5. El ndice de abortos como complicacin de la tc-
nica es del 5%.

5. Cul de las siguientes es correcta respecto a las
diferencias entre las vas sensitivas medulares dor-
sales y las anterolaterales?

1. Las dorsales cruzan al lado opuesto de la mdula
nada ms entrar por las races dorsales.
2. Las dorsales se decusan en el bulbo y suben, for-
mando parte del lemnisco medial, hasta el tlamo.
3. La transmisin por las vas anterolaterales es mu-
cho ms rpida.
4. Las anterolaterales presentan mayor grado de
orientacin espacial respecto a su origen en la
superficie del cuerpo.
5. Las anterolaterales no sufren decusacin alguna.

6. Slo uno de los siguientes efectos del sistema ner-
vioso simptico se produce al actuar sobre los
receptores beta 1:

1. Vasoconstriccin.
2. Relajacin del tero.
3. Glucogenolisis.
4. Aumento de la frecuencia cardiaca.
5. Vasodilatacin.

7. Al producto del volumen original de un vaso por
su distensibilidad se le denomina:

1. Adaptabilidad.
2. Conductancia.
3. Importancia.
4. Reclutamiento.
5. Seltbewetsein.

8. Al hablar de la circulacin pulmonar, es incorrecta
una de las siguientes afirmaciones:

1. La reduccin de la concentracin de O2 en el al-
veolo motiva una vasoconstriccin de los vasos
vecinos (efecto opuesto al de la circulacin sist-
mica).
2. En posicin erecta la presin arterial pulmonar es
unos 23 mm.Hg mayor en las bases que en los
vrtices, siendo el flujo pulmonar el triple o el
cudruple.
3. En posicin erecta, para que el ventrculo derecho
pueda bombear sangre a los vrtices pulmonares
debe vencer una presin de 15 mm.Hg en contra
(fuerza gravitatoria de la columna de sangre).
4. EN condiciones normales el gradiente alveolo ar-
terial de oxgeno obedece a la circulacin bron-
quial y a cierto grado de cortocircuito fisiolgico
intracardiaco (venas de Tebesio).
5. Las resistencias vasculares pulmonares estn regu-
ladas principalmente por el sistema nervioso vege-
tativo.

9. La saturacin habitual de la hemoglobina por
oxgeno en la sangre venosa es del:

1. 92%.
2. 97%.
3. 75%.
@
u
I
a

N
e
p
t
u
n
o
Centro de Estudios Aula Neptuno - GRANADA Ver.: 9/2001
www.aulaneptuno.com
- 2 -
4. 60%.
5. 20%.

10. La fractura ms frecuente del acetbulo es :

1. Pared posterior.
2. Columna posterior.
3. Pared anterior.
4. Columna anterior.
5. Transfondo.

11. El tto. de la enfermedad de Osgoood Schalater en
la infancia suele consistir inicialmente en:

1. Debe ser quirrgico.
2. Inmovilizacin enyesada durante una semana.
3. Reposo relativo con restriccin discreta de la acti-
vidad deportiva temporal.
4. Rodillera de entraje rotuliano.
5. Inmovilizacin enyesada durante un mes.

12. El signo del "bostezo" en abduccin de la pierna
ocurre por la lesin de:

1. LCA.
2. LCP.
3. LLE.
4. LLI.
5. LLE + LCP.

13. La "triada desgraciada" de O'Donnoghue se pre-
senta por la lesin de:

1. LCA + LLI.
2. LCA + LLI + ME.
3. LCP + LLI + MI.
4. LCP + LLE + ME.
5. LCA + LLI + MI.

14. La mayora de las fracturas de los metacarpianos
se tratan con:

1. Osteosntesis.
2. Movilizacin precoz.
3. Sntesis con agujas.
4. Traccin.
5. Reduccin cerrada e inmovilizacin.

15. En las fracturas diafisarias del hmero, la lesin
asociada ms frecuente es:

1. La trombosis de la arteria humeral.
2. La rotura de la arteria humeral.
3. La neuropraxia del nervio radial.
4. La lesin del nervio mediano.
5. La rotura de la porcin larga del biceps.

16. Rinne negativo en oido derecho y positivo en el
izquierdo y Weber lateralizado a oido derecho se
dan en:

1. Sujeto normal.
2. Hipoacusia perceptiva derecha.
3. Hipoacusia perceptiva izquierda.
4. Hipoacusia conductiva derecha.
5. Hipoacusia conductiva izquierda.

17. Cul es falsa para el colesteatoma?

1. Tratamiento quirrgico.
2. En el colesteatoma congnito, la membrana tim-
pnica suele estar intacta.
3. Es una neoplasia epitelial maligna.
4. Se produce por migracin del epitelio del CAE al
oido medio.
5. Puede producier parlisis facial.

18. La prueba de Fowler y el test S.I.S.I. se usan para
el estudio de:

1. Inteligibilidad.
2. Potenciales evocados.
3. Umbral en tonos puros.
4. Reclutamiento.
5. Fatiga patolgica.

19. El tumor parotdeo que ms frecuentemente pue-
de producir parlisis facial es:

1. Adenocarcinoma qustico.
2. Mucoepidermoide.
3. Adenoma pleomorfo.
4. Tumor de Wharton.
5. Carcinoma indiferenciado

20. FALSO para enfermedades vestibulares:

1. MENIERE - Hipoacusia de percepcin.
2. Nistagmo perifrico - Horizonto-rotatorio.
3. Neuronitis vestibular - Hipoacusia coclear.
4. Neurinoma del N. acstico - Hipoacusia retroco-
clear.
5. MENIERE - Sintomatologa vegetativa.

21. La necrosis cutnea inducida por cumarnicos a
veces se relaciona con:

1. Dficit de antitrombina III.
2. Dficit de protena C.
3. Dficit de protena S.
4. Dficit de plasmingeno.
5. Disfibrinogenemias.

22. Una prolongacin del tiempo de protrombina no
es esperable en uno de los siguientes trastornos:

1. Dficit del factor II.
2. Dficit del factor VIII.
3. Afibrinogenemia.
4. Dficit de factor VII.
5. Dficit de factor V.

23. Respecto a los grupos sanguneos, es falsa una de
las siguientes frases:

1. Los anticuerpos del sistema AB0 suelen ser aglu-
tininas naturales de tipo IgM.
2. Los genes secretores controlan la existencia o no
de sustancias del grupo sanguneo AB0 en las se-
creciones.
3. El sistema de mayor importancia en las transfu-
siones es el Rh.
4. Los anticuerpos del sistema P suelen ser naturales,
de clase IgM y actan a baja temperatura.
5. Se han descrito anticuerpos anti-N en personas
sometidaas a dilisis.

24. La principal diferencia entre el Von Willebrand y
el Bernard-Soulier estriba en que en este ltimo:

1. No hay historia familiar de hemorragias.
2. No se producen hemorragias graves.
@
u
I
a

N
e
p
t
u
n
o
Centro de Estudios Aula Neptuno - GRANADA Ver.: 9/2001
www.aulaneptuno.com
- 3 -
3. Las alteraciones no se corrigen administrando
plasma normal.
4. La agregacin en presencia de ristocetina es nor-
mal.
5. Las plaquetas son de tamao normal.

25. Mujer de 30 aos con antecedentes de dos abortos
previos. Tras el segundo de ellos se demostr la
presencia de anticoagulante lpico en su suero.
Qu tratamiento indicaras para prevenir un
nuevo aborto en su prximo embarazo?

1. Cumarnicos y AAS, 350 mg/24 horas.
2. AAS, 500 mg/24 horas.
3. Heparina de bajo peso molecular, 5000 unida-
des/24 h.
4. Prednisona, 1 mg/kg/da y AAS 80 mg/24 h.
5. Ninguna, en sus circunstancias el embarazo est
contraindicado.

26. Paciente de 76 aos con anemia, signos de hema-
topoyesis ineficaz, monocitosis y esplenomegalia.
Causa ms probable:

1. Leucemia mielomonoctica crnica.
2. Mononucleosis infecciosa.
3. S.de Van den Berghe.
4. Anemia refractaria sideroblstica.
5. Leucemia linftica crnica.

27. Causa ms frecuente de insuficiencia cardiaca en
un nio con anemia severa, VCM muy disminuido
y dianocitos con ROE elevada:

1. Por la anemia.
2. Valvulopata artica asociada.
3. Hemosiderosis.
4. Cardiopata congnita asociada.
5. Yatrognica.

28. Una de las siguientes alteraciones electrolticas se
ha descrito como una frecuente complicacin del
tratamiento sustitutivo con vitamina B12. Indcala:

1. Hipercalcemia.
2. Hipopotasemia.
3. Hipomagnesemia.
4. Hipersideremia.
5. Hiponatremia.

29. La aparicin de signos de fibrosis pulmonar en un
paciente un ao despus de recibir un trasplante
de mdula sea suele deberse a:

1. Neumona por CMV.
2. Rechazo crnico.
3. Toxicidad de la azatioprina.
4. Enf.injerto contra husped crnica.
5. Metstasis de la enfermedad que motiv el tras-
plante.

30. Seala la incorrecta. El dgto.diferencial del mielo-
ma mltiple se tiene en cuenta en:

1. Fracturas patolgicas.
2. Sndrome anmico.
3. Uremia.
4. Elevacin de PTH en suero.
5. Amiloidosis.

31. Ante un componente monoclonal en suero, Cul
de los siguientes hallazgos hace pensar que nos en-
contramos ante una gammapata monoclonal be-
nigna?

1. Aparicin de ms del 20% de clulas plasmticas
en mdula sea.
2. Creatinina elevada.
3. Hipercalcemia.
4. Velocidad de sedimentacin y hemoglobina nor-
males.
5. Hiperviscosidad.

32. En un joven de 28 aos que hemos remitido para
estudio anatomo patolgico por sospecha de En-
fermedad de Hodgkin, nos llega el siguiente infor-
me: "En los ganglios estudiados se han encontrado
bandas de fibrosis colgena que atraviesan la ar-
quitectura ganglionar, delimitndola a modo de
ndulos. Se observa infiltracin por clulas mono-
nucleares que, fijadas en formol, presentan una re-
traccin de su citoplasma, quedando espacios la-
cunares a su alrededor". El diagnstico del cuadro
es:

1. Enfermedad de Hodgkin, tipo esclerosis nodular.
2. Enfermedad de Hodgkin, tipo deplecin linfocti-
ca.
3. EH, tipo celularidad mixta.
4. EH, tipo predominio linfoctico.
5. Linfoma histioctico difuso.

33. La observacin en un paciente con esferocitosis
hereditaria de varias masas paravertebrales en la
radiografa de trax puede significar ms proba-
blemente:

1. La asociacin con una leucemia aguda no linfo-
blastica.
2. La asociacin con neuroblastoma.
3. La asociacin con timoma.
4. La presencia de eritropoyesis extramedular.
5. La presencia de infartos seos.

34. Una joven nos muestra un hematoma espectacu-
lar en la cara interna del muslo izquierdo que, se-
gn dice inicialmente, apareci de forma espont-
nea esa maana. Al preguntarle si recuerda algn
traumatismo pasado en esa zona, cuenta que hace
cuatro das su novio le dio un pellizco un poco ms
salvaje de la cuenta. Cree haber odo algo sobre
familiares suyos que tenan problemas de sangra-
dos y hematomas, as como que se han producido
muchos abortos espontneos, tanto en la familia de
su padre como en la de su madre. El estudio bsico
de la coagulacin y hemostasia en general es nor-
mal. Qu prueba diagnstica sera la ms ade-
cuada?

1. Biopsia cutnea y estudio del material con inmu-
nofluorescencia.
2. Determinacin de la agregacin plaquetaria en
presencia de ristocetina.
3. Medicin del tiempo de disolucin del cogulo en
urea 5-molar.
4. Ionotest (test del sudor).
5. Test de Rorschach al novio.

35. Los tres criterios mayores de policitemia vera
segn el P.V. study group, actualmente de gira
mundial de lanzamiento de su nuevo disco Pruri-
ginous after shower , son:
@
u
I
a

N
e
p
t
u
n
o
Centro de Estudios Aula Neptuno - GRANADA Ver.: 9/2001
www.aulaneptuno.com
- 4 -

1. Reduccin de los niveles de eritropoyetina srica,
aumento del recuento de hemates y esplenomega-
lia.
2. Esplenomegalia, saturacin arterial de oxgeno
superior al 92% y aumento de la masa eritrocitaria.
3. Aumento de la saturacin arterial de oxgeno, eri-
tropoyetina baja y aumento del valor hematocrito.
4. Aumento del recuento del hemates y del hemato-
crito, saturacin arterial de oxgeno superior al
92% y esplenomegalia.
5. Eritropoyetina baja, aumento de la masa eritrocita-
ria y esplenomegalia.

36. Un nio presenta signos de HTc secundaria a
hidrocefalia, con alteraciones visuales, DI y calcifi-
caciones supraselares en la Rx de crneo y retraso
en la edad sea. Cul es el diagnstico ms pro-
bable?

1. Craneofaringioma.
2. Meningioma.
3. Glioma del nervio ptico.
4. Tumor germinal.
5. Aneurisma de cartida interna.

37. Con respecto a la presin del LCR no es cierto
que:

1. La hipercapnea aumenta la presin del LCR.
2. Las pequeas variaciones de la TA no producen
aumento significativo de la presin del LCR.
3. La elevacin del pH incrementa la presin del
LCR.
4. Los incrementos de la presin venosa elevan la
presin del LCR.
5. La presin del LCR est en relacin con el flujo y
el volumen del LCR.

38. Cul de las siguientes proposiciones en relacin
con las diferencias clnicas entre los sndromes pi-
ramidal y extrapiramidal es falsa?

1. La hipertona del sind. piramidal afecta especial-
mente la musculatura flexora de las extremidades
superiores y extensora de las extremidades inferio-
res.
2. 2. La hipertona del sind. extrapiramidal afecta
especialmente la musculatura flexora de las ex-
tremidades y el tronco.
3. Los movimientos involuntarios anormales son ca-
ractersticos del sind. extrapiramidal.
4. Los reflejos cutneo plantares son flexores en el
sind. extrapiramidal.
5. La paresia de los movimientos voluntarios es ca-
racterstica de sndrome extrapiramidal.

39. Sobre las lesiones neurolgicas focales, indica la
afirmacin incorrecta:

1. La marcha aprxica es indicativa de lesin en el
lbulo frontal.
2. Una hemianopsia homnima puede indicar lesio-
nes unilaterales en el lbulo occipital.
3. Las alucinaciones auditivas y olfatorias son tpicas
del lobulo temporal.
4. La confusin derecha-izquierda, agnosia digital,
agrafia y acalculia son los signos producto de le-
siones en el lbulo parietal no dominante.
5. Los pacientes con lesin en el lbulo parietal do-
minante presentan fenmeno de extincin de pun-
tos simtricos.

40. Un paciente de 63 aos consulta por un cuadro
progresivo de tres meses de evolucin que se inici
con cefalea hemicraneal derecha que se incremen-
taba con la tos y los estornudos y ltimamente le
despierta por las noches, acompandose de na-
seas y vmitos. En las tres ltimas semanas nota
prdida de fuerza en la extremidad superior iz-
quierda. En la exploracin se observa ligera bra-
dipsiquia, edema de papila bilateral y una discreta
paresia braquio facial izquierda. Cul de los si-
guientes procesos le parece la causa ms probable?

1. Un proceso expansivo hemisfrico derecho.
2. Una malformacin arteriovenosa hemisfrica iz-
quierda.
3. un tumor hemisfrico izquierdo.
4. Un absceso hemisfrico izquierdo.
5. Un accidente vascular cerebral evolutivo hemisf-
rico izquierdo.

41. Un paciente de 35 aos presenta bruscamente
cefalea intensa acompaada de afasia. La explora-
cin revela prdida de fuerza en hemicuerpo dere-
cho, anestesia de esa zona y marcada rigidez de
nuca. Cul sera el diagnstico de sospecha?

1. Sepsis meningoccica de comienzo fulminante.
2. Rotura de absceso cerebral.
3. Migraa complicada con hemiplejia.
4. Hemorragia cerebral intraparenquimatosa.
5. Rotura de aneurisma del polgono de WILLIS.

42. La clsica triada de Charcot de la esclerosis mlti-
ple consta de:

1. Monoparesia, nistagmo y ceguera.
2. Nistagmo, palabra escndida y temblor intencio-
nal.
3. Dismetra, ataxia e hipoestesia.
4. Cefalea, ceguera y hemiparesia.
5. Diplopia, vrtigo y abolicin de reflejos.

43. Elige la falsa. Los signos cardinales de la Enferme-
dad de Parkinson son:

1. Rigidez en rueda dentada.
2. Hipoestesia distal.
3. Inestablidad postural.
4. Temblor de reposo.
5. Bradicinesia.

44. Un paciente de 75 aos de edad, encontrndose
previamente bien, presenta de forma aguda agita-
cin e ideas delirantes. La exploracin muestra un
paciente desatento, desorientado en tiempo y espa-
cio, con lenguaje incoherente y severa afectacin
de la memoria inmediata, pero leve de la memoria
de aprendizaje y para hechos remotos. Es capaz de
realizar rdenes sencillas y repite bien frases cor-
tas. El resto de la exploracin es normal. Tiene
probablemente:

1. Una demencia, siendo la causa ms probable el
Alzheimer.
2. Un sndrome confusional agudo.
3. Una afasia sensorial.
4. Un Sndrome de Korsakoff.
5. Un ataque de pnico.

@
u
I
a

N
e
p
t
u
n
o
Centro de Estudios Aula Neptuno - GRANADA Ver.: 9/2001
www.aulaneptuno.com
- 5 -
45. Cul es la alteracin ms importante que se ob-
serva en la Enfermedad de Hallervorden Spatz?

1. Alteracin en el metabolismo del hierro con ele-
vacin importante del hierro srico.
2. Degeneracin y depsito de lipofucsina en las mo-
toneuronas del tronco de encfalo.
3. Acmulo de pigmento en el globo plido, la sus-
tancia negra y el ncleo rojo.
4. Desmielinizacin del cuerpo calloso producida
por el alcohol.
5. Atrofia neuronal cortical difusa de causa heredita-
ria.

46. La neuralgia trigeminal bilateral en un individuo
de 22 aos es un dato diagnnstico a favor de:

1. Esclerosis mltiple.
2. Esclerosis vascular carotdea.
3. Esclerosis lateral amiotrfica.
4. Esclerosis tuberosa.
5. Esclerosis sistmica.

47. Cul de las siguientes manifestaciones NO espe-
raras encontrar en una distrofia muscular de
DUCHENNE?

1. Marcha de pato.
2. Hiperlordosis lumbar.
3. Deterioro intelectual.
4. Hipotona uterina.
5. Arrtimias cardacas.

48. La asociacin de parlisis facial bilateral y edema
facial sugiere:

1. Sarcoidosis.
2. MELKERSSON-ROSSENTHAL.
3. Moebius.
4. Esclerosis mltiple.
5. Parlisis de BELL.

49. Un paciente es trado al servicio de Urgencias tras
sufrir parada cardiorrespiratoria y posterior re-
animacin. Cul de los siguientes hallazgos a la
exploracin presupone un peor pronstico de la
encefalopata anxica?:

1. Presencia de mioclonias.
2. Ojos ligeramente divergentes.
3. Desarrollo de crisis generalizadas.
4. Patrn respiratorio rtmico.
5. Pupilas arreactivas.

50. Una mujer de 30 aos, obesa, acude al servicio de
urgencias por cefalea, rigidez de nuca, visin bo-
rrosa,diplopia, nuseas y vmitos. El fondo de ojo
muestra un papiledema bilateral. En la TC es
compatible con la normalidad y mediante la pun-
cin lumbar slo se detecta un aumento de la pre-
sin. Cul ser su diagnostico de sospecha?:

1. Glioblastoma multiforme.
2. Hidrocefalia normotensiva.
3. Hipertensin intracraneal benigna.
4. Abceso cerebral.
5. Meningitis bacteriana.

51. Qu afirmacin es falsa sobre la miastenia gra-
vis?

1. En la prueba con anticolinestersicos la sustancia
ms usada es el edrophonio.
2. Si hay alteraciones tmicas asociadas se recomien-
da la timectoma.
3. La miastenia congnita consiste en una oftalmo-
pleja bilateral transitoria en nios cuya madre pa-
dece miastenia gravis.
4. Los reflejos OT son normales al principio, pero se
agotan.
5. En el 80% de los pacientes se encuentran anti-
cuerpos anti receptores de acetil colina.

52. Slo una de las siguientes no pertenece al sndrome
occipital:

1. Estereognosia.
2. Palinopsia (persistencia de la imagen visual des-
pus de que ha desaparecido el objeto).
3. Ceguera cortical.
4. Agnosia visual de los objetos.
5. Simultagnosia (incapacidad para percibir simult-
neamente todos los elementos de una escena o
paisaje).

53. Uno de los siguientes no se utiliza ni se incluye en la
actualidad en ensayos clnicos como tratamiento
de la Enfermedad de la Motoneurona del adulto:

1. Administracin intravenosa de hormona liberado-
ra de tirotropina (TRH).
2. Administracin intratecal de TRH.
3. Interfern gamma.
4. Factor neurotrpico ciliar.
5. Factor de crecimiento I similar a la insulina (IGF-
I).

54. De los siguientes, el tumor ms frecuente del Sis-
tema Nervioso Central es:

1. Metstasis del cncer de pulmn.
2. Meningioma.
3. Meduloblastoma.
4. Astrocitoma qustico de cerebelo.
5. Astrocitoma maligno.

55. Un paciente que ha consultado por cefalea y ataxia
presenta una masa en hemisferio cerebeloso dere-
cho compatible con un tumor. Se aprecia rigidez
de nuca y papiledema. EL diagnstico ms proba-
ble es:

1. Meningioma.
2. Papiloma del plexo coroideo.
3. Pinealoma.
4. Glioblastoma multiforme.
5. Meduloblastoma.

56. No es criterio de causalidad:

1. Multicausalidad.
2. Efecto dosis-respuesta.
3. Evidencia experimental.
4. Plausibilidad biolgica.
5. Secuencia temporal.

57. La probabilidad de encontrar un resultado positi-
vo en un test cuando el paciente tiene la enferme-
dad es del 70 %. A qu medida de la validez de
un test pertenece este dato?

1. Especificidad.
@
u
I
a

N
e
p
t
u
n
o
Centro de Estudios Aula Neptuno - GRANADA Ver.: 9/2001
www.aulaneptuno.com
- 6 -
2. Sensibilidad.
3. VPP.
4. VPN.
5. Riesgo relativo.

58. Si un test que se aplica a la poblacin da resultado
positivo en 70 de cada 100 enfermos, y da resulta-
do negativo en 50 de cada 100 sanos, cul ser su
valor predictivo positivo?

1. 50%.
2. 70%.
3. 58%.
4. 62%.
5. No se puede calcular con los datos de que dispo-
nemos.

59. Si dos pases tienen igual tasa bruta de mortalidad
y el pas A tiene una poblacin ms joven que el
pas B, significa que en el pas B:

1. Hay peores condiciones sanitarias.
2. Hay menor TAE.
3. Hay mayor TAE.
4. Hay una guerra civil.
5. La expectativa de vida ser menor que en el pas
A.

60. Mediante la "razn de los productos cruzados"
podemos estimar:

1. Incidencias.
2. Fraccin etiolgica.
3. Riesgo relativo.
4. Riesgo atribuible.
5. Ninguno de los anteriores.

61. De los siguientes valores, cul es el ms til admi-
nistrativamente?

1. Fraccin etiolgica poblacional.
2. Riesgo relativo.
3. Riesgo atribuible poblacional.
4. Fraccin etiolgica.
5. Riesgo atribuible.

62. El porcentaje de casos de una enfermedad en
individuos expuestos, que se deben al factor de
riesgo, se llama:

1. Prevalencia en expuestos.
2. Fraccin etiolgica poblacional.
3. Incidencia en expuestos.
4. Riesgo atribuible.
5. Fraccin etiolgica.

63. Cul de las siguientes afirmaciones relativas a los
estudios experimentales es falsa?

1. Proporcionan un mayor control del factor de estu-
dio.
2. Permiten generalizar de forma fcil y amplia sus
resultados.
3. Son los que proporcionan la mayor evidencia de
relacin causa-efecto.
4. Suelen tener un coste elevado, aunque ello depen-
de de la duracin y complejidad del protocolo.
5. Restricciones ticas impiden que muchas pregun-
tas puedan ser abordadas siguiendo su metodolo-
ga.

64. Si decimos que la diferencia de riesgos (o "diferen-
cia de incidencias" o "riesgo atribuible") de
EPOC en los fumadores es del 10%, estamos refi-
rindonos a:

1. El 10% de los fumadores desarrollan EPOC.
2. El 10% de los fumadores desarrollan EPOC por
culpa del hbito de fumar.
3. El 10% de los casos de EPOC en los fumadores
obedecen al hbito de fumar.
4. El 10% de los casos de EPOC ocurre en indivi-
duos fumadores.
5. El 90% de los casos de EPOC en los fumadores
ocurren por el hbito de fumar.

65. Qu parmetro determina la magnitud de una
asociacin en un estudio epidemiolgico e indica la
probabilidad de que una enfermedad se desarrolle
en el grupo expuesto con relacin al no expuesto?

1. Riesgo atribuible.
2. Fraccin etiolgica.
3. Riesgo relativo.
4. Incidencia.
5. Prevalencia.

66. En un estudio de cohortes que pretende estudiar el
aumento del riesgo de infarto agudo de miocardio
que produce el consumo de anticonceptivos orales
se han seguido 10.000 mujeres en edad frtil du-
rante diez aos, obtenindose en el anlisis epide-
miolgico un riesgo relativo de 2,35. cmo inter-
pretaras este resultado?

1. La diferencia de riesgo de desarrollar IAM entre
las mujeres que usan anticonceptivos orales res-
pecto a las que no los usan es de 2,35 tras diez
aos de seguimiento.
2. Una mujer que utilice anticonceptivos orales tiene
2,35 veces ms riesgo de desarrollar un IAM en
diez aos que una mujer que no los utilice.
3. El porcentaje de casos atribuibles a la utilizacin
de anticonceptivos orales, tras un seguimiento de
diez aos, es de 2,35%.
4. La proporcin de casos de IAM en las mujeres
que durante diez aos utilizaron anticonceptivos
orales y que son debidos a la utilizacin de stos es
de 2,35%.
5. La proporcin de casos de IAM entre las mujeres
que emplearon o no anticonceptivos orales es de
2,35%.

67. No interviene en la patogenia de la insuficiencia
renal aguda:

1. Difusin transtubular del filtrado.
2. Reduccin del flujo sanguneo renal.
3. Obstruccin tubular por cilindros.
4. Aumento del coeficiente de ultrafiltracin.
5. Reduccin de la permeabilidad de la membrana
basal glomerular.

68. En una IRA ser raro encontrar uno de los si-
guientes datos analticos:

1. Potasio: 6,6 mEq/L.
2. pH: 7,58.
3. Calcio: 7,5 mg/dL.
4. Hb.: 8,2 g./dL.
5. Urea: 71 mg/dL.

@
u
I
a

N
e
p
t
u
n
o
Centro de Estudios Aula Neptuno - GRANADA Ver.: 9/2001
www.aulaneptuno.com
- 7 -
69. El desarrollo en un paciente con artritis gotosa de
una insuficiencia renal lentamente progresiva, con
riones atrficos e hipertensin, debe considerarse
en primer lugar el diagnstico de:

1. Nefropata por plomo.
2. Amiloidosis renal.
3. Diabetes nefrognica familiar.
4. Rin poliqustico del adulto.
5. Hidronefrosis.

70. Una de las siguientes sustancias aumenta el flujo
sanguneo renal:

1. Acetilcolina.
2. Clorotiacida.
3. Angiotensina II.
4. Acido acetil saliclico.
5. Noradrenalina.

71. Nia de once aos, que dos das despus de una
infeccin farngea presenta edemas generalizados.
Expl.fsica: Palidez cutneo-mucosa, edemas,
TA:180-90. Analtica: Hb.: 9.6 g/dL, urea 74
mg/dL, protenas totales 5.3 g/dL, creatinina 2.1
mg/dL. Sedimento de orina: Proteinuria de 5
gramos en 24 horas, 15-20 hemates por campo.
Analtica especial: Descenso de las fracciones del
complemento C1q, C2, C3 y C4. Biopsia renal:
Proliferacin mesangial, MBG gruesa con depsi-
tos granulares subendoteliales de C3, C4, proper-
dina e IgG. Diagnstico:

1. GN.postestreptoccica.
2. Nefropata mesangial IgA.
3. GN.mesangiocapilar tipo II.
4. Hialinosis focal y segmentaria.
5. GN.mesangiocapilar tipo I.

72. En cul de las siguientes causas de alcalosis meta-
blica hay una reduccin de los cloruros en orina?

1. Sndrome de Bartter.
2. Consumo abusivo de diurticos.
3. Vmitos subrepticios.
4. Hiperaldosteronismo primario.
5. Sndrome de Cushing.

73. Una mujer de 49 aos, diagnosticada de poliquis-
tosis renal, y con una creatinina srica de 3 mg/dl,
acude a urgencias con dolores en el abdomen y en
el costado. Dice que el da anterior la orina sala te-
ida de sangre. En la ecografa se observan riones
poliqusticos y ecos muy complejos en el polo supe-
rior del rin derecho. La causa ms probable de
este cuadro es:

1. Un infarto renal.
2. Una infeccin urinaria.
3. Un carcinoma de clulas renales.
4. Una hemorragia de un quiste renal.
5. Una malformacin arteriovenosa en el rin.

74. Ante una infeccin urinaria con hematuria, sn-
drome cisttico, fiebre alta con alteracin del estado
general y puopercusin renal positiva, el diagns-
tico ms probable es:

1. Prostatitis.
2. Pielonefritis aguda.
3. Cistitis asociada a litiasis.
4. Perihepatitis post uretritis.
5. Cistitis intersticial.

75. Los padres de un nio de 18 meses consultan por-
que ste presenta un cuadro de vmitos, poliuria y
falta de ganancia ponderal desde hace varias se-
manas. Se comprueba en la analtica de sangre un
pH srico de 7,276, un potasio de 2,8 mEq/L, un
bicarbonato de 20 mEq/L y un cloro de 112
mEq/L. Por radiografa se observa litiasis clcica
en ambos riones y en orina hay hipercalciuria y
un pH urinario elevado. Qu prueba de entre las
siguientes puede tener ms utilidad en la confir-
macin diagnstica de este caso?

1. Medicin de aldosterona en respuesta a la admi-
nistracin de solucin salina intravenosa.
2. Test de tolerancia oral a la glucosa.
3. Determinacin de la excrecin fraccional del bi-
carbonato.
4. Determinacin de la presin parcial de CO2 urina-
rio tras forzar una diuresis alcalina.
5. Determinacin de PTH tras administracin de
hidrocortisona intravenosa.

76. Paciente de 37 aos que consulta por dolor lumbar
y s. cisttico. En la exploracin se constata HTA y
hematuria en el sedimento. Interrogado el enfer-
mo, refiere antecedentes de hemorragia subarac-
noidea 2 aos atras as como habar sido diagnosti-
cado de diverticulosis intestinal. La primera prue-
ba diagnstica que realizara es:

1. Estudio de 5-HIA urinario.
2. Determinacin de catecolaminas plasmticas.
3. TAC craneal.
4. Clculo del hematocrito.
5. Ecografa renal.

77. Una de las relaciones detalladas a continuacin
entre tumor renal y factor predisponente no es co-
rrecta:

1. Transicional de la pelvis- Analgsicos.
2. Nefroblastoma-Sturge-Weber-Krabbe.
3. Adenocarcinoma-Von Hippel Lindau.
4. Hamartoma-Esclerosis tuberosa.
5. Adenocarcinoma-Tabaco.

78. Seala la falsa en el cncer de prstata:

1. El 95% son adenocarcinomas.
2. Las metstasis ms frecuentes son las de los hue-
sos de la pelvis.
3. La fosfatasa cida es su marcador bioqumico ms
sensible.
4. El tratamiento de los casos metastatizados es la
hormonoterapia.
5. La prostatectoma radical es el tratamiento de
eleccin del estadio B.

79. Cul de los siguientes frmacos es la causa ms
frecuente de cistitis qumica?

1. Ciclofosfamida.
2. Ciclosporina.
3. Metotrexato.
4. Azatioprina.
5. Suero antilinfocitario.

80. La composicin litisica ms frecuente es:
@
u
I
a

N
e
p
t
u
n
o
Centro de Estudios Aula Neptuno - GRANADA Ver.: 9/2001
www.aulaneptuno.com
- 8 -

1. Oxalato clcico.
2. Fosfato clcico.
3. Fosfato amnico magnsico.
4. Cistina.
5. Acido rico.

81. Indica la correspondencia adecuada entre infec-
ciones del tracto urinario inferior y la clnica que
provocan:

1. La prostatitis crnica bacteriana cursa con infec-
ciones urinarias de repeticin o sntomas de hiper-
trofia prosttica benigna en fase inicial. Al tacto
rectal la prstata es prcticamente normal.
2. En la cistitis hay disuria, polaquiuria, urgencia
miccional y enuresis en nios, pero no aparece ni
dolor hipogstrico ni hematuria.
3. Se define la abacteriuria sintomtica por la presen-
cia de urocultivos positivos sin sintomatologa de
vas urinarias.
4. Chlamydia trachomatis es la causa ms frecuente
de orquiepididimitis aguda en el anciano.
5. En la prostatitis aguda bacteriana aparece un sn-
drome cisttico con dolor sacro o perineal, afebril y
con prstata muy inflamada al tacto rectal.

82. Con respecto a los distintos tipos de incontinencia
urinaria, sus caractersticas y posible tratamiento,
indica la afirmacin incorrecta:

1. Entre los ancianos, la incontinencia es ms fre-
cuente en los de ms edad, en el sexo masculino y
en los de hbito sedentario.
2. La incontinencia DE URGENCIA va precedida
de deseos sbitos e imperiosos de orinar con im-
posibilidad de parar el reflejo miccional. Se produ-
ce en el prostatismo, inflamaciones de la pared ve-
sical y enfermedades neurolgicas y en el trata-
miento mdico pueden usarse anticolinrgicos, an-
tagonistas del calcio, antidepresivos tricclicos o al-
fa bloqueantes.
3. La incontinencia DE ESTRES aparece con la tos,
la risa, estornudos y otros esfuerzos que aumenten
la presin abdominal; se favorece por trastornos
posicionales vesicoesfinterianos y uterinos (partos
laboriosos) y se pueden usar los alfa adrenrgicos
y los antidepresivos tricclicos.
4. La incontinencia POR REBOSAMIENTO o pa-
radjica aparece en la neuropata autnoma (veji-
ga neurgena) o el prostatismo y hay incontinen-
cia nocturna al relajarse el sistema esfinteriano.
Pueden usarse los alfa bloqueantes y los agonistas
colinrgicos.
5. La incontinencia verdadera o neurgena es la que
aparece en la mayora de las enfermedades neuro-
lgicas (tumores, ACV, esclerosis multiple ) y
se manifiesta como una prdida intermitente e im-
previsible de orina.

83. Cul de las siguientes actitudes es ms correcta
ante un paciente que acude al Centro de Salud con
parafimosis ("Dcese de la complicacin de la fi-
mosis consistente en una estrangulacin del glande
por el cuello prepucial que, una vez retrado por
detrs del surco balnico, no puede volverse a su
posicin normal")?

1. Derivar a Urgencias del Hospital para circuncisin
(prostectoma).
2. Recomendar calma al enfermo y decirle que el
problema se soluciona solo, pero que si en dos d-
as no se ha reducido debera acudir al especialista.
3. Intentar la reduccin manual tras compresin del
glande durante cinco minutos para reducir su ta-
mao.
4. Tratamiento con diacepam intramuscular y pautar
antihistamnicos durante dos semanas.
5. Negarnos a atenderle e increparle por acudir a un
Centro del venereable Servicio Andaluz de Salud
en ese estado.

84. Una afirmacin sobre el Tumor de Wilms es falsa:

1. La presentacin bilateral se observa entre el 5 y el
10% de los casos, simultnea o sucesivamente.
2. Se han descrito casos asociados a aniridia,
hemihipertrofia, disgenesia gonadal y retraso men-
tal.
3. Se puede diagnosticar fcilmente por ecografa al
comprobarse una masa slida con calcificaciones
excntricas en su interior, casi patognomnicas.
4. No suele dar lugar a metstasis seas.
5. En el 5-10% de los casos puede existir anulacin
funcional del rin por obstruccin de vas, por
invasin de pedculo o por sustitucin del parn-
quima renal por tejido tumoral.

85. De los siguientes datos comparativos entre una
situacin de cetoacidosis diabtica (CAD) y una de
coma hiperglucmico hiperosmolar no cetsico
(CHHNC), indica el incorrecto:

1. La hipopotasemia es ms importante en el
CHHNC que en la CAD.
2. La deshidratacin es mayor en el CHHNC que en
la CAD.
3. El CHHNC es menos frecuente que la CAD.
4. La afeccin de la conciencia es ms grave en el
CHHNC que en la CAD.
5. La etiologa infecciosa es ms frecuente en el
CHHNC que en la CAD.

86. Cul de los siguientes compuestos farmacolgicos
es capaz de negativizarr las glucosurias detectadas
por el mtodo de la tira reactiva?

1. Acido nalidxico.
2. Salicilatos.
3. Cefalosporinas.
4. Probenecid.
5. Acido ascrbico.

87. La lesin renal ms frecuentemente encontrada en
la diabetes mellitus es:

1. Glomeruloesclerosis intercapilar difusa.
2. Glomeruloesclerosis nodular (KIMMELSTIEL -
WILSON).
3. Necrosis papilar.
4. Infeccin urinaria.
5. Nefritis intersticial crnica.

88. En relacin a los estudios etiopatognicos sobre la
DM NO es cierto:

1. La presencia de HLA DQw7 se ha relacionado
con una mayor incidencia de DMID.
2. En la DMID existe < incidencia de transmisin a
familiares que en la DMNID.
3. Se han detectado anomalas en el cromosoma 11
en el caso de DMNID.
@
u
I
a

N
e
p
t
u
n
o
Centro de Estudios Aula Neptuno - GRANADA Ver.: 9/2001
www.aulaneptuno.com
- 9 -
4. La DM tipo MODY tiene herencia AD.
5. La presencia de Amilina en islotes pancreticos se
relaciona con mayor incidencia de DMNID.

89. Un varn de 68 aos con diabetes mellitus tipo II,
de aproximadamente tres aos de evolucin, en
buen control metablico con dieta y, al parecer, no
hipertenso (en un control realizado dos aos antes),
presenta TA 220/120, encontrndose como nicos
hallazgos en la exploracin fsica galope por cuarto
tono y frecuentes cruces arteriovenosos en el fondo
de ojo. La radiografa de abdomen muestra asime-
tra de tamao entre las siluetas renales, siendo la
derecha de unos 8 cm.de largo. Qu dgto.debe
sospecharse en primer lugar?

1. Trombosis de la vena renal derecha.
2. Nefropata diabtica.
3. Pielonefritis crnica unilateral.
4. Estenosis arteriosclertica de la arteria renal dere-
cha.
5. Uropata obstructiva.

90. El dficit de xantina oxidasa se relaciona con una
de las siguientes manifestaciones:

1. Hiperuricemia e hiperuricosuria.
2. Artritis aguda.
3. Nefropata intersticial.
4. Nefrolitiasis.
5. Depsitos de cristales de urato monosdico en car-
tlagos.

91. Un paciente con vmitos intensos de repeticin
puede padecer ms probablemente uno de los si-
guientes trastornos analticos:

1. Hiponatremia, hipopotasemia, acidosis metabli-
ca.
2. Hipernatremia, hipopotasemia, acidosis metabli-
ca.
3. Hiponatremia, hiperpotasemia, alcalosis metabli-
ca.
4. Hiponatremia, hiperpotasemia, acidosis metabli-
ca.
5. Hiponatremia, hipopotasemia, alcalosis metabli-
ca.

92. Cul de las siguientes variedades de porfiria NO
puede incluirse bajo el trmino "porfirias hepti-
cas agudas"?

1. Coproporfiria hereditaria.
2. Porfiria aguda intermitente.
3. Porfiria variegata.
4. Porfiria congnita de GNTHER.
5. Porfiria de DOSS.

93. Una de las siguientes apoprateinas es de sintesis
intestinal:

1. Apo B-100.
2. Apo B-48.
3. Apo C-I.
4. Apo C-II.
5. Apo E.

94. Joven de 15 aos ingresado por neumona bacte-
riana. En la anamnesis destaca la presencia de fre-
cuentes epstaxis y gingivorragias, astenia, dolores
seos generalizados y ausencia de trastornos neu-
rolgicos. A la exploracin hay hepatomegalia, es-
plenomegalia marcada y edemas maleolares. Ana-
lticamente hay pancitopenia y en la puncin de
mdula sea se encuentran unas clulas con varios
ncleos pequeos y excntricos y citoplasma "en
seda arrugada", con lisosomas cargados de gluco-
cerebrsido. En los leucocitos hay una disminucin
de la actividad glucocerebrosidasa. Dgto:

1. Leucodistrofia metacromtica.
2. Enf.de Niemann-Pick.
3. Enf.de Gaucher tipo I.
4. Enf.de Tay-Sachs.
5. Enf.de Gaucher tipo III.

95. Qu afirmacin es falsa sobre la psoriasis pustu-
losa generalizada?

1. En la forma aguda, la piel aparece intensamente
enrojecida.
2. El etretinato est contraindicado en esta forma de
psoriasis.
3. Se ha descrito su asociacin con el HLA B27.
4. Entre los factores desencadenantes figuran la
hipocalcemia, infecciones y frmacos.
5. Los pacientes estn predispuestos a sepsis por
St.aureus.

96. Cul es la mejor opcin teraputica en un pacien-
te con Sndrome de Sweet en el que no pueden uti-
lizarse los corticoides?

1. Metotrexate.
2. Indometacina.
3. Yoduro potsico.
4. Hidroxicloroquina.
5. Dapsona.

97. La causa farmacolgica ms frecuente de pnfigo
es:

1. D-penicilamina.
2. Captopril.
3. Indometacina.
4. Propranolol.
5. Rifampicina.

98. Cul de estos anticuerpos no suele encontrarse en
las personas con dermatitis herpetiforme?

1. Anti gliadina.
2. Anti endomisio.
3. Anti reticulina.
4. Anti clulas parietales gstricas.
5. Anti membrana basal epidermo-drmica.

99. La transformacin ms frecuente de una querato-
sis actnica es hacia:

1. Melanoma de extensin superficial.
2. Melanoma nodular.
3. Epitelioma espinocelular.
4. Epitelioma basocelular.
5. Lntigo maligno melanoma.

100. En el estudio histolgico de un melanoma se mide
la distancia en milmetros desde la clula granulosa
ms alta hasta la tumoral ms profunda, evalun-
dose en 1,34 milmetros. En la clasificacin de
Breslow, ello representa un estadio:

@
u
I
a

N
e
p
t
u
n
o
Centro de Estudios Aula Neptuno - GRANADA Ver.: 9/2001
www.aulaneptuno.com
- 10 -
1. 1.
2. 2.
3. 3.
4. 4.
5. 5.

101. En el tratamiento de la A.R., uno de estos trata-
mientos se emplea cuando han fracasado los otros.
Selalo:

1. Metilprednisolona.
2. Aurotiomalato sdico.
3. Sulfasalacina.
4. D-penicilamina.
5. Diclofenaco.

102. Nia de 8 aos con artritis de rodilla e iridociclitis.
Tambin tendr:

1. Anticuerpos antinucleares.
2. HLA B27.
3. VSG acelerada.
4. Factor reumatoide.
5. Exantema maculopapuloso.

103. En la definicin diagnstica de la enfermedad
mixta del tejido conjuntivo se pueden utilizar crite-
rios que incluyen el ttulo elevado de anticuerpos
anti ribonucleoprotena y las cinco manifestaciones
clnicas ms frecuentes en el citado sndrome. En-
tre stas no se encuentra:

1. Sinovitis.
2. Miositis.
3. Esclerodactilia.
4. Pericarditis.
5. Fenmeno de Raynaud.

104. Durante 15 aos, una mujer de 50 aos de edad ha
presentado fenmeno de Raynaud de las manos.
El padecimiento ha empeorado durante el ltimo
ao, y han aparecido artralgias y artritis que afec-
tan manos y muecas, as como esclerodactilia leve
y dificultad para deglutir alimentos slidos. Los es-
tudios de laboratorio revelan positividad de anti-
cuerpos antinuclleares sricos a una dilucin de
1:160. El dgto. ms probable es:

1. Esclerosis sistmica progresiva.
2. Enfermedad mixta del tejido conjuntivo.
3. Sndrome de superposicin.
4. Dermatomiositis.
5. Lupus eritematoso sistmico.

105. A continuacin te presentamos dos casos clnicos
cuyo diagnstico es similar. Indicalo. a) Un sujeto
de 25 aos de edad ha tenido dolor e inflamacin
en la rodillla derecha durante el ltimo ao. Por lo
dems est bien y no hay antecedentes de trauma-
tismo. Las radiografas muestran varias erosiones
en el borde de la articulacin de la rodilla derecha.
Al aspirar la articulacin se obtienen 25 ml. de l-
quido sinovial oscuro parduzco con buena viscosi-
dad. b) Un hombre de 28 aos presenta desde
hace ocho meses dolor e inflamacin en la rodilla
derecha sin haber sufrido ningn tratamiento pre-
vio. La artrocentesis realizada en tres ocasiones di-
ferentes ha permitido extraer un lquido sinovial
sanguinolento no coagulable; el recuento de leuco-
citos ha sido inferior a 1000/mm3. Los cultivos de
bacterias, micobacterias y hongos dieron resultado
negativo. Por lo dems, el paciente se encuentra
bien y no tiene antecedentes de ditesis hemorrgi-
ca. En la exploracin fsica se detecta una inflama-
cin moderada en la rodilla derecha, sin dolor a la
palpacin; el resto de la exploracin es normal. La
rodilla es estable y tiene un rango de movimientos
casi completo. Se realiz una artrografa, que fue
diagnstica: El dgto.de estos dos pacientes es:

1. Artritis reumatoide atpica.
2. Sndrome de Reiter.
3. Hemangioma.
4. Osteocondritis disecante.
5. Sinovitis vellonodular pigmentada.

106. Una afirmacin es falsa referente a la fiebre medi-
terrnea familiar:

1. El 50% de los pacientes no tienen historia familiar
de al enfermedad.
2. La amiloidosis es una complicacin comn en to-
do el mundo, y no existen diferencias geogrficas
para su presentacin.
3. La colchicina es probable que sea beneficiosa.
4. Los niveles de dopamina-beta hidroxilasa es pro-
bable que estn elevados.
5. El dolor torcico es frecuente.

107. Cul de las siguientes es FALSA en la E. de PA-
GET?

1. Se inicia con dolor seo, aunque ms frecuente-
mente es asintomtica.
2. La afectacin vertebral da imagen en vertebra de
marfil.
3. Se han observado inclusiones intranucleares en os-
teoclastos.
4. La malignizacin en osteosarcoma se produce en
un 10%.
5. Se asocia a la presencia de condrocalcinosis.

108. Cul es el dato radiolgico ms tpico de la artritis
psorisica?

1. Erosin del ngulo anterosuperior del disco inter-
vertebral.
2. Esclerosis subcondral en la rodilla.
3. Resorcin en las falanges distales.
4. Deformidad en las metacarpofalngicas.
5. Crneo en sal y pimienta.

109. Ricardo C. es un joven de 30 aos que consulta
por intenso dolor en las articulaciones de rodilla,
tobillo y pie. Tiene conjuntivitis, estomatitis indolo-
ra y lesiones papuloescamosas y pstulas en pal-
mas y plantas. Como antecedentes destaca un
cuadro diarreico con vmitos dos semanas antes
de la consulta. Hacia dnde debe ir dirigida nues-
tra siguiente exploracin?

1. Al tracto respiratorio superior.
2. A la columna lumbar.
3. Al tracto digestivo.
4. A la sensibilidad de las extremidades superiores.
5. A la parte pudenda.

110. Para el diagnstico de la Granulomatosis de We-
gener la presencia de dos ms criterios propor-
ciona una sensibilidad del 88% y una especificidad
del 92%. Cul no pertenece a esos criterios?

@
u
I
a

N
e
p
t
u
n
o
Centro de Estudios Aula Neptuno - GRANADA Ver.: 9/2001
www.aulaneptuno.com
- 11 -
1. Inflamacin nasal u oral; lceras orales con dolor o
sin l, o secrecin nasal purulenta o hemorrgica.
2. Presencia en la piel de ppulas eritematosas cuyo
color no desaparece a la vitropresin.
3. Alteraciones de la radiografa de trax: ndulos,
cavitacin o infiltrados no migratorios ni fugaces.
4. Alteraciones en el sedimento urinario:
microhematuria o cilindros hemticos.
5. Presencia de infiltracin granulomatosa en la
biopsia: inflamacin granulomatosa dentro de la
pared de una arteria o en la regin perivascular o
extravascular de una arteria o arteriola.

111. Uno de los siguientes no es criterio diagnstico de
la arteritis temporal:

1. Cefalea de aparicin reciente o caractersticas dis-
tintas a las habituales.
2. Alteraciones visuales (amaurosis, diplopia, visin
borrosa ).
3. Mejora evidente en las primeras 48 horas de tra-
tamiento con glucocorticoides.
4. Biopsia de arteria temporal positiva.
5. Duracin de los sntomas inferior a una semana.

112. La radiografa de trax en el estadio I de la sarcoi-
dosis suele mostrar:

1. Patrn reticulonodulillar bilateral de predominio
en lbulos superiores.
2. Patrn bilateral en vidrio deslustrado, ms acusado
en lbulos superiores.
3. Patrn alveolar con broncograma areo en uno de
los pulmones.
4. Adenopatas hiliares bilaterales sin infiltrados
pulmonares.
5. Absolutamente nada, ni siquiera esternn, costillas
o corazn.

113. Mujer de 20 aos que con fines autolticos ingiere 5
horas antes 20 gr. de paracetamol .Es traida a ur-
gencias presentando un cuadro de nauseas y dolor
abdominal. Cul es su tratamiento especfico?:

1. Difenilhidantoina i.v.
2. Manitol i.v. a dosis altas
3. N-acetilcisteina
4. Oxifenacetina
5. Dosis altas de furosemida

114. Un varon de 32 aos es llevado a Urgencias tras
una ingesta copiosa de lejia concentrada con fines
autoliticos. Aqueja dolor epigstrico, hematemesis
de sangre roja y se objetiva neumoperitoneo en el
estudio radiologico. Cul es, entre las actuaciones
propuestas, la que debe realizarse en primer lu-
gar?:

1. Laparotomia urgente
2. TAC abdominal
3. TAC toracica
4. Panendoscopia oral urgente
5. Administracion de agua albuminosa fria

115. Paciente con herida en muslo derecho, en principio
tratada como herida contaminada, con sutura
primaria, y que dos das despus presenta dolor in-
tenso, edema, exudado y linfangitis. El paciente es-
t febril. Cul de estas intervenciones no est jus-
tificada?

1. Escisin en bloque, extirpando tejido necrtico y
parte de tejido sano, para obtener un buen sangra-
do que depure la regin.
2. Vaciamiento de colecciones purulentas y apertura
de espacios residuales.
3. Desbridamiento enzimtico con varidasa o cola-
genasa.
4. Sutura secundaria a las dos-tres semanas.
5. Reposo, antibiticos y drenaje postural.

116. En un paciente con orquiepididimitis, en un prin-
cipio no est indicado utilizar:

1. Antibiticos si se sospecha infeccin bacteriana.
2. Analgsicos.
3. Drenaje.
4. Suspensorios.
5. Anti-inflamatorios.

117. Afirmacin falsa en las hernias inguinales:

1. La hernia no reductible y con compromiso vascu-
lar se denomina estrangulada.
2. Las hernias crurales tienen estrecha relacin con
los vasos femorales.
3. El saco herniario lo constituye el peritoneo parie-
tal.
4. Las hernias inguinales directas son las que se pro-
ducen a travs del conducto inguinal.
5. Las hernias crurales pasan por debajo del ligamen-
to inguinal.

118. Cul de las siguientes manifestaciones es ms
rara en las hernias de hiato por deslizamiento?

1. Incarceracin.
2. Esofagitis.
3. Asociacin con colelitiasis.
4. Reflujo gastro-esofgico.
5. Asociacin con diverticulosis.

119. Con objeto de calcular la extensin de quemadu-
ras, es necesario tener en cuenta que la distribu-
cin de la superficie corporal en las regiones ana-
tmicas del cuerpo en nios difiere de la corres-
pondiente en adultos. Cules de las afirmaciones
siguientes acerca del rea de superficie en nios es
vlida en comparacin con los adultos?

1. La de las extremidades inferiores es relativamente
ms grande, y la de cabeza y cuello, relativamente
menor.
2. La de las extremidades inferiores es relativamente
ms grande y la de las extremidades superiores un
poco menor.
3. La de las extremidades inferiores es menor, y la de
cabeza y cuello, ms grande.
4. La de las extremidades inferiores es relativamente
menor, en tanto la de extremidades superiores es
un poco ms grande.
5. La de todas las extremidades es considerablemen-
te menor, en tanto la de cabeza y cuello es prcti-
camente similar.

120. Paciente que acude a urgencias por agresin con
arma blanca a nivel epigstrico. En la exploracin
abdominal presenta signos de peritonismo y peris-
taltismo ausente. La actitud en este caso sera:

1. Exploracin de la herida y observacin.
2. Puncin lavado peritoneal.
@
u
I
a

N
e
p
t
u
n
o
Centro de Estudios Aula Neptuno - GRANADA Ver.: 9/2001
www.aulaneptuno.com
- 12 -
3. Laparotoma exploradora.
4. Radiografa simple y, si existiera neumoperitoneo,
laparotoma exploradora.
5. Ecografa abdominal para detectar la presencia de
lquido libre intraperitoneal.

121. El tratamiento de eleccin de la apnea del sueo
obstructiva es:

1. Oxigenoterapia en gafas nasales.
2. CPAP.
3. Antidepresivos.
4. Broncodilatadores.
5. Corticoides.

122. El primer factor fisiopatolgico en la gnesis del
cor pulmonale suele ser:

1. Hipertrofia del v.derecho.
2. Alteracin en el intercambio de gases.
3. Vasoconstriccin de la arteria pulmonar.
4. Crecimiento de la A.derecha.
5. Prdida del lecho vascular pulmonar.

123. Un paciente afecto de artritis reumatoide en tra-
tamiento con penicilamina y que en su vida ha vis-
to una mina (salvo las del lpiz), presenta un cua-
dro de dos meses de evolucin consistente en tos no
productiva y disnea de esfuerzo. Se auscultan es-
tertores inspiratorios difusos. En la radiografa de
trax existe insuflacin pulmonar con pequeas
opacidades difusas y refuerzo de la trama bronco-
vascular. En el estudio funcional respiratorio se
diagnostica un trastorno obstructivo y en el lavado
broncoalveolar intensa neutrofilia. Cul de los si-
guientes te parece el diagnstico ms probable en
este paciente?

1. Pleuritis tuberculosa.
2. Enfisema pulmonar avanzado.
3. Bronquiolitis obliterante.
4. Fibrosis pulmonar idioptica.
5. Sndrome de Kaplan.

124. Cul de los siguientes tratamientos no es til en
un paciente que acude a urgencias con una crisis
asmtica grave?

1. Nedocromilo sdico subcutneo.
2. Oxigenoterapia en mascarilla al 35%.
3. Nebulizacin de betamimticos.
4. Teofilina intravenosa.
5. Metilprednisolona intravenosa.

125. Un varn de 50 aos de edad presenta pancreatitis
relacionada con el paso de un clculo biliar. Su tra-
tamiento incluye meperidina y solucin salina por
va intravenosa. Dos das ms tarde presenta an-
siedad, taquipnea y disnea. Una radiografa de t-
rax de urgencia demuestra infiltrados intersticiales
y alveolares bilaterales difusos. Hace un ao sufri
un IAM, pero desde entonces no tiene signos de in-
suficiencia cardiaca congestiva. En este caso el sn-
drome de dificultad respiratoria del adulto puede
distinguirse mejor del edema pulmonar cardige-
no por:

1. Radiografa de trax.
2. Gasometra arterial.
3. Medicin de la presin en cua pulmonar.
4. Medicin de la adaptabilidad pulmonar.
5. Clculo de la diferencia alveolo-arterial de O2.

126. Tras instaurar tratamiento con aerosoles de ago-
nistas beta a un paciente con un ataque agudo de
asma, cul de estos datos indica que se puede
mantener dicho tratamiento y no es necesario aso-
ciar otras medidas?

1. Aumento del uso de la musculatura accesoria.
2. Disminucin del flujo pico.
3. Elevacin de la PCO2 hasta lmites normales.
4. Disminucin del pulso paradjico.
5. VEMS inferior al 20% del previsto y que no se
duplica en la primera hora de tratamiento.

127. La afectacin pulmonar menos frecuente entre las
siguientes, en la fibrosis qustica, es:

1. Bronquiectasias.
2. Obstruccin al flujo areo.
3. Hipoxemia.
4. Infecciones recurrentes sinopulmonares.
5. Derrame pleural

128. En el lavado broncoalveolar de un paciente con
sarcoidosis es tpico un incremento en la propor-
cin de:

1. Polimorfonucleares.
2. Macrfagos.
3. Eosinfilos.
4. Linfocitos B.
5. Linfocitos T.

129. La enfermedad cptica es una alveolitis alrgica
extrnseca cuyo antgeno es:

1. Los botones de las gabardinas de color beige.
2. Los crepes de bechamel y gambas.
3. Las anillas de las latas de aceitunas rellenas de an-
choas.
4. Los compact disc de Mara del Monte.
5. Las envolturas de las momias.

130. Rogelia H. de 35 aos fue diagnosticada de quera-
toconjuntivitis seca hace dos meses. Ahora consul-
ta por tos seca y disnea. Existe un patrn intersti-
cial bilateral difuso en la radiografa de trax y en
el lavado alveolar hay abundantes linfocitos. Se
comprueba en la electroforesis de protenas sricas
una gammapata policlonal. El diagnstico ms
probable es:

1. Fibrosis de Hamman Rich.
2. Neumona intersticial linfoidea.
3. Neumona eosinfila crnica.
4. Sndrome de Caplan.
5. Bronquiolitis obliterante.

131. En un estudio hemodinmico pulmonar, un pa-
ciente presenta una presin sistlica de arteria
pulmonar de 54 mm.Hg., y una presin capilar
pulmonar de 13 mm.Hg. Qu etiologa explica es-
tos resultados?

1. Fibrosis pulmonar difusa.
2. Enfermedad venooclusiva.
3. Hipoventilacin alveolar central.
4. Sndrome de la colza.
5. Hipertensin pulmonar primitiva.

@
u
I
a

N
e
p
t
u
n
o
Centro de Estudios Aula Neptuno - GRANADA Ver.: 9/2001
www.aulaneptuno.com
- 13 -
132. No suele producir parlisis bilateral del diafrag-
ma:

1. Cancer pulmonar.
2. Enf. Duchenne.
3. ELA.
4. Guillain-Barr.
5. Neuropata alcohlica.

133. Uno de estos datos no suele estar producido por los
neuroblastomas mediastnicos:

1. Disfona.
2. Ptosis.
3. Dolor interescapular.
4. Disfagia.
5. Miosis.

134. Si slo nos dan el siguiente dato de un paciente:
"Presenta un derrame pleural con elevacin de
amilasa de origen salival", el diagnstico ms pro-
bable es:

1. Rotura esofgica.
2. Sialolitiasis.
3. Sndrome de Meigs.
4. Seudoquiste pancretico.
5. Carcinoma de pncreas.

135. Cul es la causa ms frecuente de Eosinofilia
pulmonar? :

1. Frmacos.
2. Helmintos.
3. Neumona eosinfila crnica.
4. Enf. de Churg-Strauss.
5. Aspergilosis Broncopulmonar Alrgica.

136. Paciente con sospecha de cncer broncopulmonar,
que aqueja ataxia, sensacin de inestabilidad, v-
mitos y prdida de fuerza muscular. La estirpe
implicada en dicho cncer probablemente es:

1. Epidermoide.
2. Oat cell.
3. Adenocarcinoma.
4. Clulas grandes.
5. Linfoma.

137. Una de las siguientes interpretaciones de las mani-
festaciones clnicas del cncer broncopulmonar es
incorrecta. Selala:

1. La aparicin de citopenias y eritroblastosis hace
pensar en invasin de la mdula sea.
2. El dolor en el hombro irradiado a la regin cubital
del brazo indica extensin local de un tumor del
vrtice pulmonar.
3. La aparicin de periostitis y dedos en palillo de
tambor son indicativos de una enfermedad para-
neoplsica.
4. Los sndromes de compresin medular se deben a
la extensin regional del tumor.
5. Puede observarse hiperfuncin corticosuprarrenal
como sndrome paraneoplsico y, ms raramente,
hipofuncin corticosuprarrenal por metstasis.

138. El principal factor protector frente al edema pul-
monar crnico en ausencia de cambios del tono
vascular pulmonar es:

1. El gran aumento del drenaje a cargo del sistema
venoso linftico.
2. La presin negativa intersticial.
3. La presin positiva intersticial.
4. El aumento de la presin capilar pulmonar.
5. La reduccin de la presin osmtica en la cpsula
de Bowman.

139. Una es falsa respecto a la hiperclaridad pulmonar
unilateral:

1. La etiologa de la enfermedad corresponde real-
mente a una bronquiolitis obliterante adquirida du-
rante la infancia a partir de una infeccin (vrica o
por Mycoplasma)
2. Los espacios alveolares presentan enfisema pana-
cinar de carcter moderado, lo que explica la hi-
perinsuflacin.
3. La arteria pulmonar ipsilateral es pequea y tiene
pocas ramificaciones.
4. La enfermedad es habitualmente asintomtica y se
descubre por casualidad en una radiografa de t-
rax.
5. En la mayora de los casos la afectacin es bilate-
ral.

140. Cunto tiempo tardan en desaparecer, en perso-
nas previamente sanas, las imgenes radiogrficas
de una neumona?

1. 3 das.
2. Una semana.
3. Dos semanas.
4. Cuatro semanas.
5. Tres meses.

141. Los factores de necrosis tumoral se pueden consi-
derar como pertenecientes al grupo de la interleu-
kina:

1. 1.
2. 2.
3. 3.
4. 4.
5. 5.

142. Cul de estos no es un rgano linfoide, fuera del
periodo fetal?

1. Mdula sea.
2. Hgado.
3. Timo.
4. Sistema MALT.
5. Bazo.

143. Seala la asociacin FALSA en relacin a las in-
munoglobulinas :

1. Ig M - nica que acta como antitoxina.
2. Ig D - activa la va alternativa.
3. Ig G - principal Ig que interviene en la CCDA.
4. Ig E - la Fc se une a los mastocitos.
5. Ig A - su componente secretor se sintetiza en las
cl. epiteliales.

144. Mujer de 35 aos que consulta por ndulo nico
en el lbulo tiroideo derecho; en la gammagrafa se
ve un ndulo fro (hipocaptante). Las hormonas ti-
roideas son normales. Qu actitud es la ms ade-
cuada?

@
u
I
a

N
e
p
t
u
n
o
Centro de Estudios Aula Neptuno - GRANADA Ver.: 9/2001
www.aulaneptuno.com
- 14 -
1. Tiroidectoma total puesto que el ndulo fro es la
forma de presentacin del cncer de tiroides.
2. Tiroidectoma total seguida de dosis ablativa de
radioyodo.
3. PAAF.
4. Tratamiento supresor con T4.
5. Revisiones peridicas.

145. En la produccin de cul de estos efectos puede
estar implicado el VIP?

1. Hipocortisolismo.
2. Crisis asmtica.
3. Ulcera duodenal.
4. Aumento de la contractilidad cardiaca.
5. Diabetes mellitus.

146. En relacin a la secrecin de hGH es falso que:

1. La hipoglucemia aumenta sus niveles.
2. Tiene un factor hipotalmico estimulador.
3. El factor inhibidor hipotalmico es la somatostati-
na.
4. La seccin del tallo hipofisario produce aumento
en sangre.
5. Los glucocorticoides inhiben su liberacin.

147. Una mujer embarazada acude al mdico porque
dice estar ms nerviosa y haber notado que suda
ms que antes y ocasionales palpitaciones. La fre-
cuencia cardiaca es de 100 lpm. y la glndula tiroi-
des est ligeramente agrandada. El estudio funcio-
nal tiroideo muestra: T3 y T4 totales elevadas con
ndice de T3 libre y de T4 libre dentro de la nor-
malidad. El diagnstico ms probable es:

1. Enfermedad de Graves.
2. Tiroiditis indolora linfoctica.
3. Aumento de la concentracin de TBG.
4. Sndrome de Refetoff.
5. Tiroiditis de De Quervain.

148. Si un paciente presenta un cuadro que evoluciona
en las siguientes fases: 1 Poliuria y polidipsia; 2
Aumento de la osmolaridad urinaria; 3 Nueva
aparicin de poliuria y polidipsia, que ya perma-
necen de forma continua, la causa que mejor ex-
plica estos hallazgos es:

1. Seccin del tallo hipofisario.
2. S.de Wolfram.
3. Destruccin de los ncleos supraptico y paraven-
tricular, con conservacin de la neurohipfisis.
4. Destruccin de los ncleos hipotalmicos y la neu-
rohipfisis.
5. Destruccin exclusiva de la neurohipfisis.

149. Cul de las siguientes modalidades de tratamien-
to elegira ante un paciente de 68 aos, cardipata,
con bocio multinodular hiperfuncionante?:

1. Tiroidectoma subtotal.
2. Antitiroideos ms tiroidectoma subtotal.
3. I-131.
4. Antitiroideos ms I-131.
5. Antitiroideos nicamente.

150. Cul de las siguientes pautas es mejor para tratar
a un paciente diagnosticado de feocromocitoma?

1. Propranolol solamente.
2. Propranolol seguido de fenoxibenzamina.
3. Fenoxibenzamina seguida de propranolol.
4. Prazosn solamente.
5. Propranolol seguido de prazosn.

151. Cul de las siguientes situaciones puede aumen-
tar los niveles de CBG o transcortina?

1. Sndrome nefrtico.
2. Enf.de Menetrier.
3. Diabetes mellitus.
4. Hipotiroidismo.
5. Cirrosis heptica.

152. De los siguientes frmacos capaces de producir un
sndrome de Cushing, seala aqul cuya accin se
ejerce a nivel suprarrenal:

1. Ciproheptadina.
2. Metergolina.
3. Metopirona.
4. Valproato.
5. Bromocriptina.

153. La posibilidad de aparicin de una neoplasia en
una paciente con sndrome de TURNER, est ms
directamente relacionada con:

1. Edad de aparicin de las manifestaciones clnicas.
2. Mosaicismo con cromosoma Y.
3. Infecciones intercurrentes.
4. Antecedentes familiares.
5. Presencia de cintillas gonadales.

154. Una mujer joven acude a nuestra consulta refi-
riendo astenia, prdida de peso e hiperpigmenta-
cin progresiva. El estudio funcional de la paciente
muestra un cortisol libre urinario bajo, ausencia
de respuesta del cortisol srico al estmulo de
ACTH y un incremento en la eliminacin urinaria
de catecolaminas tras estmulo con 2-deoxi-D-
glucosa. Cul ser probablemente la causa de su
insuficiencia suprarrenal?:

1. Tuberculosis.
2. SIDA.
3. Metstasis.
4. Amiloidosis.
5. Adrenalitis autoinmune.

155. Con relacin a los sndromes pluriglandulares
inmunes, uno de los siguientes datos es ms propio
del tipo I que del tipo II S. de SCHMIDT:

1. Suele afectar a adultos.
2. Diabetes mellitus.
3. Candidiasis.
4. Enfermedad de ADDISON.
5. Tiroiditis linfocitaria.

156. Se hace un estudio de fertilidad en un hombre de
35 aos. Se encuentra que tiene un recuento es-
permtico bajo en tres eyaculaciones diferentes.
La FSH plasmtica es de 10 UI/l. La biopsia testi-
cular es normal. La etiologa ms probable para la
infecundidad de este hombre es :

1. Enf. hipofisaria.
2. Alt. adquirida de la espermatognesis.
3. Enf. hipotalmica.
4. Antecedentes de orquitis viral.
@
u
I
a

N
e
p
t
u
n
o
Centro de Estudios Aula Neptuno - GRANADA Ver.: 9/2001
www.aulaneptuno.com
- 15 -
5. Bloqueo del conducto deferente.

157. Una persona con hipercalcemia debido a Sarcoi-
dosis tendra todo lo siguiente MENOS :

1. Rx anormal de trax.
2. Absorcin aumentada del cacio en el aparato di-
gestivo.
3. Hipercalciuria.
4. Aumento de la concentracin srica de hormona
paratiroidea.
5. Hipergammaglobulinemia.

158. EN la insuficiencia corticosuprarrenal secundaria
no es cierta una de estas proposiciones:

1. No suele manifestarse por hiperpigmentacin.
2. EL tratamiento consiste en la administracin de
prednisona y fludrocortisona.
3. La respuesta del cortisol a la administracin de
ACTH est disminuida.
4. No es frecuente encontrar hiperpotasemia.
5. La secrecin basal de ACTH suele estar dismi-
nuida o inadecuadamente normal.

159. Con respecto al tratamiento con frmacos neuro-
loticos, una no es correcta:

1. Puede producir amenorrea.
2. Los de mayor potencia son ms cardiotxicos.
3. Pueden disminuir el umbral convulsivgeno.
4. Las distonas agudas aparecen en las primeras fa-
ses del tratamiento.
5. La hipotensin ortosttica puede producirse desde
los primeros das del tratamiento.

160. Alguna de las siguientes no indica buen pronstico
en la esquizofrenia:

1. Inicio tardo de la enfermedad.
2. Presencia de factores precipitantes.
3. Comienzo insidioso.
4. Predominio de sntomas productivos.
5. Presencia de sntomas afectivos.

161. No es tpica del trastorno pasivo-agresivo:

1. Accesos de ira inmotivados cuando tiene que estar
desocupado.
2. Postergacin de las tareas, de modo que nunca las
realiza en los plazos fijados.
3. Evitacin de las obligaciones, alegando olvidos.
4. Crtica de forma irracional a las personas con car-
gos de autoridad.
5. Trabaja deliberadamente despacio o mal cuando
hace tareas que no quiere hacer en realidad.

162. Cuando una persona se autoprovoca lesiones con
intencin de enfermar y de ser atendido como un
enfermo, se llama:

1. S.de Cottard.
2. S.de Bricquet.
3. S.de Mnchausen.
4. S.de Capgras.
5. S.de Ganser.

163. Cul de las siguientes sustancias puede provocar
crisis de angustia en pacientes con antecedentes de
crisis de angustia espontneas?

1. Antidepresivos.
2. Alfa metil-dopa.
3. Haloperidol.
4. Clonacepam.
5. Cafena.

164. La doble orientacin suele darse en una de las
siguientes patologas:

1. Histeria conversiva.
2. Esquizofrenia.
3. Demencia senil.
4. Demencia multiinfarto.
5. Personalidades esquizoides.

165. La estructura anatmica con mayor importancia
en los procesos de memoria y atencin es:

1. Corteza del lbulo parietal.
2. Hipotlamo.
3. Hipocampo.
4. Ncleos del rafe.
5. Amgdala.

166. La amnesia antergrada suele observarse sobre
todo en:

1. La totalidad de los traumatismos craneales.
2. Sndrome de Korsakoff.
3. Pacientes ansioso-depresivos.
4. Histerias disociativas.
5. Pacientes con dj vu.

167. Una estudiante universitaria de 21 aos de edad
apareci con su madre en un consultorio mdico
quejndose de que se haba despertado haca dos
das con adormecimiento y parlisis total de am-
bas piernas. No saba lo que le pasaba, pero haba
hecho venir a su madre, que resida en La Toja,
para que cuidara de ella. Durante el ltimo ao
haba compartido el apartamento con un amigo,
pero despus de muchas broncas, l se march el
da antes al inicio de los sntomas. A la exploracin
era totalmente incapaz de mover ambas piernas y
experimentaba anestesia total y falta de respuesta
a los estmulos dolorosos (pinchazos) en ambas
piernas hasta las ingles, en las que se recuperaba
sbitamente la sensibilidad. Todos los reflejos eran
rigurosamente normales, as como el resto de la
exploracin fsica. La alteracin debe calificarse
como:

1. Trastorno dismrfico.
2. Simulacin simple.
3. Trastorno por conversin.
4. Sndrome de Meadows.
5. Sndrome de somatizacin.

168. Un paciente que padeca un trastorno por estrs
postraumtico tras un naufragio, experimentaba
todos los siguientes sntomas propios del cuadro,
excepto uno, que no pertenece al mismo:

1. Pensaba que le quedaba poca vida por vivir.
2. Tena explosiones de ira.
3. Se notaba insensible y distante a sus familiares.
4. Olvid por completo los nombres de los compa-
eros fallecidos en el accidente.
5. Empez a sufrir automatismos verbales.

169. Una es falsa respecto al tratamiento de la farin-
@
u
I
a

N
e
p
t
u
n
o
Centro de Estudios Aula Neptuno - GRANADA Ver.: 9/2001
www.aulaneptuno.com
- 16 -
goamigdalitis estreptoccica:

1. La penicilina G es el antibitico de eleccin.
2. Previene el desarrollo de fiebre reumtica.
3. Previene las complicaciones supurativas.
4. Previene la glomerulonefritis.
5. Debe ser prolongado (10 das).

170. Cul de los siguientes grmenes es el ms frecuen-
temente responsable de la infeccin de una deriva-
cin colocada para tratar una hidrocefalia?

1. St.epidermidis.
2. St.aureus.
3. Corynebacterium diphteriae.
4. E.coli.
5. Bacteroides fragilis.

171. En cul de las siguientes entidades NO est au-
mentada la incidencia de infecciones estafiloccicas
:

1. Diabetes mellitus
2. Drepanocitosis.
3. Enf. Granulomatosa crnica.
4. Sd. de JOB.
5. Gripe.

172. Tratamiento de un portador biliar de salmonella
con litiasis biliar.

1. Colecistectoma y antibioterapia.
2. Ciprofloxacino.
3. Cotrimoxazol.
4. Colecistectoma sola.
5. Cefalosporinas.

173. Slo una de las siguientes afirmaciones sobre las
infecciones por Pseudomonas aeruginosa es inco-
rrecta:

1. La otitis evterna maligna conduce a una osteomie-
litis progresiva de la base del crneo en la que
puede provocarse meningitis, pero en la que tpi-
camente estn respetados los pares craneales.
2. Las lceras corneales constituyen una complica-
cin de las lentes de contacto, sobre todo cuando
se usan lentes de contacto blandas durante mucho
tiempo.
3. Una manifestacin caracterstica del pioderma por
Pseudomonas es su asociacin a un exudado azul-
verdoso y un olor afrutado caracterstico.
4. La infeccin crnica del tracto respiratorio inferior
por P.aeruginosa es frecuente en los enfermos con
fibrosis qustica y suele ocurrir en nios mayores o
adultos jvenes.
5. El ectima gangrenoso es la lesin cutnea espec-
fica que acompaa a la sepsis por P.aeruginosa.

174. Falsa en la Fiebre Botonosa Mediterrnea:

1. El exantema respeta palmas y plantas.
2. Est causado por la R.conorii.
3. La mancha negra no se observa en otras rickettsio-
sis.
4. En los nios la clnica es ms leve.
5. La doxicilina es eficaz en pautas cortas de un solo
da.

175. Segn el grupo de trabajo sobre tuberculosis, si en
un enfermo con dicha infeccin y en tratamiento
antituberculoso presenta, despus de haberse ob-
servado dos cultivos consecutivos negativos, la re-
aparicin de dos nuevos cultivos consecutivos posi-
tivos con nmero creciente de colonias, podemos
definirlo como:

1. Abandono teraputico.
2. Recada.
3. Fracaso teraputico.
4. Enfermo crnico.
5. Tuberculosis postprimaria.

176. Paciente VIH (+) que presenta muguet oral, disfa-
gia y pirosis retroesternal. Causa ms probable de
este cuadro:

1. CMV.
2. Candida.
3. V.Herpes Simple
4. Sarcoma de Kaposi.
5. Cryptosporidium.

177. En el SIDA, la estirpe celular implicada en las
alteraciones neurolgicas es:

1. Killer.
2. Leucocito.
3. Fibroblasto.
4. CD8.
5. Macrfago.

178. Slo una se corresponde con las infecciones por
Pn.carinii:

1. Su principal marcador bioqumico es la VSG.
2. Produce hiperplasia de los neumocitos tipo II
3. La forma epidmica se da en nios con LLA.
4. El derrame pleural suele complicarse con empie-
ma frecuentemente.
5. Suele producir una infiltracin granulomatosa
multisistmica.

179. El tratamiento prolongado con uno de los siguien-
tes frmacos predispone a la aparicin de mucor-
micosis sinusal o pulmonar. Selalo:

1. Desferrioxamina.
2. Ciclofosfamida.
3. Aurotiomalato.
4. Nitrofurantona.
5. Difenilhidantona.

180. Indica la afirmacin falsa sobre las manifestacio-
nes clnicas de la infeccin gonoccica:

1. Actualmente la epididimitis y la prostatitis son dos
complicaciones muy raras de las uretritis gonoc-
cicas.
2. La propagacin de la infeccin desde el endocr-
vix hasta las trompas de Falopio se produce, como
mnimo, en el 15% de las mujeres con infeccin
gonoccica.
3. En la salpingitis, a pesar de tratamiento antibitico
precoz, la posibilidad de afectacin permanente bi-
lateral de las trompas es casi constante, con la con-
siguiente afectacin de la fertilidad.
4. Los pacientes con infeccin gonoccica disemi-
nada suelen presentar fiebre, poliartralgias y lesio-
nes cutneas petequiales, pustulosas o necrticas
con tendencia a afectar la regin distal de los
miembros.
@
u
I
a

N
e
p
t
u
n
o
Centro de Estudios Aula Neptuno - GRANADA Ver.: 9/2001
www.aulaneptuno.com
- 17 -
5. En la infeccin gonoccica diseminada la afecta-
cin articular inicial se limita caractersticamente a
una tenosinovitis que afecta sasimtricamente a
varias articulaciones, sobre todo muecas, dedos,
rodillas y tobillos.

181. En la biopsia intestinal de un paciente joven homo-
sexual se aprecian macrfagos con estructuras
PAS positivas. Cul de las siguientes enfermeda-
des es ms probable?

1. Linfangiectasia intestinal.
2. Infeccin por Cryptosporidium.
3. Enfermedad celiaca.
4. Infeccin por Mycobacterium avium intracellulla-
re.
5. Infeccin por Giardia lamblia.

182. En la tuberculosis inicial actualmente se prefieren
utilizar las pautas de seis meses, por algunas carac-
tersticas que presentan respecto a pautas ms lar-
gas, entre las que NO se encuentra:

1. Menor coste.
2. Mejor cumplimiento.
3. Menor toxicidad.
4. Mayor tasa de resistencias adquiridas.
5. Mayor poder bactericida.

183. Qu conducta tomaras ante un joven de 15 aos
PPD negativo que vive con su padre, quien acaba
de ser diagnosticado de una tuberculosis pulmonar
activa?

1. Prescribir dos meses de tratamiento con isoniacida
y posteriormente repetir el PPD.
2. Prescribir seis meses de tratamiento con isoniaci-
da.
3. Prescribir doce meses de tratamiento con isoniaci-
da.
4. Prescribir tratamiento de seis meses con isoniaci-
da, rifampicina y piracinamida en los dos prime-
ros.
5. No es necesario hacer nada en principio. Debe re-
petirse el PPD a los dos meses y, si sale positivo,
es cuando prescribiramos la quimioprofilaxis.

184. En condiciones normales, la presin sistlica del
ventrculo derecho debe ser equivalente a:

1. Presin sistlica de VI.
2. Presin sistlica de AI.
3. PCP.
4. Presin sistlica de la arteria pulmonar.
5. Presin sistlica de vena cava superior.

185. Una mujer de 42 aos tiene edema maleolar bila-
teral de aparicin reciente. En la exploracin, el
pulso venoso en la yugular es de 4 cm.de H2O y no
hay reflujo hepatoyugular. El dgto.diferencial del
edema maleolar de esta mujer debe hacerse con
todos los procesos siguientes, excepto:

1. Tromboflebitis pelviana.
2. Varicosidades venosas.
3. Edema cclico.
4. Hipoalbuminemia.
5. Insuficiencia cardiaca derecha.

186. Cuando un enfermo se presenta con disnea y en la
auscultacin pulmonar se oyen sibilancias:

1. La insuficiencia cardiaca es un diagnstico proba-
ble.
2. La presencia de sibilancias descarta la I.C.
3. Tiene asma bronquial.
4. Nunca se deben usar diurticos.
5. Est contraindicado el uso de inhaladores de ago-
nistas beta.

187. Uno de los siguientes frmacos enlentece la con-
duccin auriculoventricular por alargamiento del
perodo refractario del nodo. Selelo:

1. Dobutamina.
2. Digital.
3. Atropina.
4. Isoproterenol.
5. Tazolol.

188. El parmetro que mejor valora el pronstico de un
IAM a largo plazo es:

1. Existencia de complicaciones.
2. Fraccin de eyeccin.
3. Extensin.
4. Localizacin.
5. ECG.

189. Un paciente de 39 aos es ingresado en UCI por
un IAM anterior extenso. Se realiza trombolisis
con rtPA, que resulta efectiva. La fraccin de eyec-
cin medida por ecocardiograma es de un 38%. A
los cuatro das de evolucin se decide su traslado al
Servicio de Cardiologa, en donde su evolucin es
satisfactoria hasta el sexto da, en que vuelve a pre-
sentar dolor precordial en reposo de 10 minutos de
duracin, y que cede tras dos comprimidos de
NTG sublingual. Durante el episodio doloroso se
registraron cambios ECG en precordiales V2 a
V5. Cul es cierta sobre este caso?

1. La aparicin de arritmias durante la reperfusin
con trombolticos es una complicacin peligrosa
que obliga a suspender dicho tratamiento.
2. La aparicin de angina post-IAM es ms frecuen-
te en los pacientes que han recibido trombolisis
que en los que no lo han hecho.
3. En este paciente no es necesario emplear nuevas
pruebas diagnsticas.
4. Al existir una depresin importante de la funcin
de VI, no est indicado el tratamiento a largo plazo
con IECA ni con beta bloqueantes.
5. El alta de la UCI fue excesivamente precoz.

190. Una mujer de 48 aos se queja de episodios de
dolor retroesternal irradiado a brazos desde hace
varios aos. El dolor aparece tanto en relacin con
esfuerzos importantes como en reposo, y dura
unos 5 minutos. No es obesa, diabtica ni hiperten-
so, y su nico factor de riesgo coronario es el humo
del tabaco, ya que, aunque no fuma, es propietaria
de un bar desde los 20 aos y reconoce que el am-
biente siempre est muy cargado. El ECG en re-
poso es normal. Se obtiene un registro ECG-
Holter, en el cual se registra un episodio de angor
de esfuerzo, con descenso de S-T, y una crisis noc-
turna de dolor durante la cual se observa una ele-
vacin del S-T. Qu afrimacin es cierta con res-
pecto a este caso?

1. Lo ms probable es que no existan lesiones obs-
@
u
I
a

N
e
p
t
u
n
o
Centro de Estudios Aula Neptuno - GRANADA Ver.: 9/2001
www.aulaneptuno.com
- 18 -
tructivas en el rbol coronario de la enferma.
2. La causa ms probable de la clnica anginosa en
este caso es el sndrome X.
3. Con toda probabilidad, la paciente ser candidata a
la realizacin de una coronariografa.
4. Es casi imposible que la paciente, con los antece-
dentes referidos, tenga cardiopata isqumica.
5. Est contraindicado realizar una ergometra en este
tipo de pacientes.

191. Un hombre de 68 aos, que ha tenido recientemen-
te un episodio sincopal, es hospitalizado con un
cuadro de insuficiencia cardiaca congestiva. Su
presin arterial es de 160/80 mm.Hg, la frecuencia
del pulso, de 80 latidos por minuto y hay un soplo
sistlico rudo de grado III/VI. El ecocardiograma
muestra un tabique interventricular despropor-
cionadamente grueso y un movimiento anterior de
la vlvula mitral durante la sstole. Cul de las si-
guientes manifestaciones es ms probable que exis-
ta en este enfermo?

1. Irradiacin del soplo hacia las cartidas.
2. Disminucin del soplo al apretar los puos.
3. Retraso de la porcin ascendente del carotidogra-
ma.
4. Disminucin de la fraccin de eyeccin del ventr-
culo izquierdo.
5. Signos de estenosis mitral.

192. Todas las cardiopatas congnitas que se mencio-
nan, salvo una, provocarn un cortocircuito iz-
quierda-derecha, en general sin cianosis:

1. Origen anmalo de la arteria coronaria izquierda a
partir del tronco de la pulmonar.
2. Conducto arterioso permeable sin hipertensin
pulmonar.
3. Comunicacin anmala total de las venas pulmo-
nares.
4. Defecto del tabique interventricular.
5. Comunicacin interauricular con defecto del tipo
seno venoso.

193. En todos los procesos siguientes, excepto en uno,
podra observarse por cateterismo cardiaco una
disminucin del gasto cardiaco, una elevacin de
las presiones diastlicas finales, derecha e izquier-
da, y un trazado con un declive y una meseta en la
porcin diastlica del registro de la presin ventri-
cular:

1. Distrofia muscular de Duchenne.
2. Amiloidosis.
3. Hemocromatosis.
4. Sndrome hipereosinoflico.
5. Fibrosis endomiocrdica.

194. Uno de los siguientes signos no es caracterstico de
la pericarditis constrictiva:

1. Hepatomegalia y edemas.
2. Ingurgitacin yugular muy llamativa.
3. Ascitis.
4. Ruido o chasquido pericrdico a la auscultacin
cardiaca.
5. Roce pericrdico a la auscultacin cardiaca.

195. Qu cardiopata congnita se asocia con ms
frecuencia al sndrome de Turner?

1. Comunicacin interventricular.
2. Estenosis pulmonar.
3. Persistencia del conducto arterioso.
4. Coartacin de la aorta.
5. Tetraloga de Fallot.

196. En un paciente de 54 aos con estenosis artica, y
que consulta por angina de esfuerzo, se diagnostica
por ECO-doppler una estenosis severa con gra-
diente transvalvular de 87 mm.Hg. Nuestra acti-
tud debe ser:

1. Eco transesofgico.
2. Indicacin de ciruga de sustitucin valvular.
3. Coronariografa previa a la decisin quirrgica.
4. Tratamiento conservador dada la edad del pacien-
te.
5. Ergometra maximal.

197. El dato ms valioso a la hora de distinguir entre el
ostium primum y un defecto secundario del tabi-
que auricular es:

1. El soplo sistlico de eyeccin en BEI.
2. El segundo ruido cardiaco.
3. El aumento de la silueta de la aurcula derecha en
la Rx.de trax.
4. La desviacin del eje elctrico a la izquierda en el
ECG.
5. La presencia de fibrilacin auricular.

198. En el diagnstico diferencial de la insuficiencia
artica, cul de estos datos es propio de las for-
mas crnicas?

1. Aumento de la fraccin de eyeccin.
2. Ausencia de cardiomegalia.
3. PTDVI muy elevada.
4. Disminucin del gasto cardiaco.
5. Soplo diastlico precoz corto.

199. En la ciruga arterial directa, la endarterectoma
consiste en:

1. Extraccin de trombos frescos formados in situ.
2. Extraccin de cogulos.
3. Extirpacin de placa de ateroma.
4. Dilatacin de zona de estenosis mediante catter
con baln.
5. Colocacin de prtesis con estructura de malla.

200. Cuando en la toma de presin arterial el odo dis-
tingue el primer sonido arterial se llama:

1. Fase 1 de Korotkoff.
2. Fase 2 de korotkoff.
3. Fase 3 de Korotkoff.
4. Fase 4 de Korotkoff.
5. Fase 5 de Smirnoff.

201. Todas las siguientes condiciones se encuentran
estrechamente relacionadas con la aparicin de
hipertensin arterial, excepto:

1. Edad y sexo.
2. Consumo de sal y de grasas.
3. Estrs y ruido ambiental.
4. Alcohol.
5. Tabaco y caf.

202. El criterio aceptado actualmente para definir la
@
u
I
a

N
e
p
t
u
n
o
Centro de Estudios Aula Neptuno - GRANADA Ver.: 9/2001
www.aulaneptuno.com
- 19 -
hipertensin arterial en la poblacin adulta (de 14
a 60 aos), es:

1. La media de dos determinaciones de TAD es igual
o superior a 85 mm.Hg.
2. La media de dos determinaciones de TAD es igual
o superior a 90 mm.Hg., o la media de dos deter-
minaciones de TAS es igual o superior a 140
mm.Hg.
3. La media de dos determinaciones de TAD es igual
o superior a 159 mm.Hg.
4. La media de dos determinaciones de TAS es igual
o superior a 160 mm.Hg.
5. La media de dos determinaciones de TAD es igual
o superior a 95 mm.Hg. o la media de dos deter-
minaciones de TAS es igual o superior a 160
mm.Hg.

203. La mayora de las muertes causadas por la hiper-
tensin arterial se deben a:

1. Nefroesclerosis hialina.
2. Nefroangiosclerosis.
3. Hiperuricemia.
4. Infarto de miocardio o insuficiencia cardiaca con-
gestiva.
5. Encefalopata hipertensiva.

204. Indica la falsa sobre la fisiopatologa de la Estenosis
Artica:

1. En pacientes con EA de larga evolucin el gasto
cardiaco puede estar conservado, a pesar de existir
un marcado gradiente transvalvular, gracias a la
hipertrofia ventricular izquierda.
2. El segundo ruido es dbil debido a la reduccin
del flujo de sangre a travs de la vlvula artica es-
tentica.
3. Un nmero importante de pacientes con EA tiene
valvulopata mitral asociada; as, en pacientes con
Insuficiencia Mitral la EA aumenta considerable-
mente la gravedad de aqulla.
4. En pacientes con EA, con y sin estenosis coronaria
asociada, se pueden encontrar datos metablicos
de isquemia miocrdica.
5. La prdida de una enrgica contraccin auricular
en el tiempo apropiado, como ocurre en la fibrila-
cin auricular y en la disociacin auriculo-
ventricular puede provocar un rpido empeora-
miento de los sntomas de EA.

205. Sobre la Insuficiencia Artica aguda, cul de las
siguientes afirmaciones no es correcta?

1. La presin del pulso arterial est ms elevada que
en las formas crnicas.
2. El soplo diastlico es, caractersticamente, breve.
3. La endocarditis bacteriana, la diseccin artica y
los traumatismos son sus causas ms frecuentes.
4. Los pacientes presentan congestin y edema pul-
monar e hipotensin secundaria al bajo gasto car-
diaco.
5. la presin diastlica ventricular aumenta de forma
considerable.

206. Indica cul de las siguientes consideraciones sobre
la fisiopatologa de la miocardiopata hipertrfica
es incorrecta:

1. Slo una cuarta parte de los pacientes con miocar-
diopata hipertrfica presentan un gradiente en la
va de salida del ventrculo izquierdo, por lo que la
principal alteracin es la disfuncin diastlica, no
la sistlica.
2. En la obstruccin de la miocardiopata hipertrfica
se produce un fenmeno dinmico: el movimiento
sistlico anterior de la vlvula mitral, hecho que no
est presente en otras obstrucciones al tracto de sa-
lida del ventrculo izquierdo.
3. En el fracaso diastlico la patogenia se explica por
un aumento en la concentracin intracelular de
calcio como consecuencia de trastornos en el ma-
nejo del mismo, que provoca hipertrofia de las fi-
bras y aumento de la rigidez de la pared ventricu-
lar.
4. El aumento de la contractilidad muscular aumenta
la velocidad de expulsin de la sangre a travs de
la va de salida, empujando a la valva mitral ante-
rior contra el tabique.
5. La disminucin de la precarga y, por tanto, del vo-
lumen ventricular, reduce ms an el tamao de la
va de salida. Tambin aumenta la obstruccin la
reduccin de la postcarga.

207. En cuanto al diagnstico diferencial entre cuatro
procesos que pueden provocar trastornos clnicos
similares: el taponamiento cardiaco, la pericarditis
constrictiva, la miocardiopata restrictiva y el in-
farto del ventrculo derecho, una de las afirmacio-
nes que siguen es errnea:

1. De las cuatro, en la que ms frecuentemente se
observa signo de Kussmaul es en la pericarditis
constrictiva.
2. En todas, menos en el IAM del ventrculo dere-
cho, se observa aumento de velocidad del flujo de
llenado diastlico precoz.
3. El espesor miocrdico slo est aumentado en la
miocardiopata restrictiva.
4. El tamao del ventrculo derecho slo est aumen-
tado en el IAM del ventrculo derecho.
5. En donde menos se esperara encontrar una des-
cendente y prominente es en el taponamiento car-
diaco; tampoco en el IAM del ventrculo derecho
es frecuente dicho hallazgo.

208. Cul de las siguientes valvulopatas cursa de
forma tpica con una onda a grande en el pulso ve-
noso yugular sin datos de hipertensin pulmonar
ni de agrandamiento del ventrculo derecho?

1. Estenosis Mitral.
2. Insuficiencia tricspide.
3. Estenosis artica.
4. Insuficiencia Mitral.
5. Estenosis Tricspide.

209. La enfermedad maligna mas frecuente en la in-
fancia es:

1. Tumor de Wilms
2. Leucemia aguda
3. Neuroblastoma
4. Meduloblastoma
5. Teratoma

210. En un recin nacido con candidiasis oral, la fuente
primaria ms frecuente de infeccin es:

1. La vagina materna.
2. Los objetos contaminados.
3. La utilizacin de NO3Ag.
@
u
I
a

N
e
p
t
u
n
o
Centro de Estudios Aula Neptuno - GRANADA Ver.: 9/2001
www.aulaneptuno.com
- 20 -
4. El contacto con portadores hospitalarios.
5. La terapia antibitica sistmica.

211. Para qu se usa el test Apt?

1. Prueba cualitativa de intoxicacin por monxido
de carbono.
2. Prueba semicuantitativa para la intoxicacin por
plomo.
3. Prueba cualitativa para la hemoglobina fetal.
4. Prueba de seleccin para la Hb.S.
5. Prueba para determinar la viscosidad samgunea.

212. En los partos difciles se producen fracturas yatro-
gnicas. Cul de estas es la ms frecuente?

1. Epifisiolisis cabeza humeral.
2. Fractura difisis humeral.
3. Fractura difisis femoral.
4. Fractura de radio.
5. Fractura de cbito y radio.

213. El enanismo en "Ratn Mickey" se describe en el
sndrome de:

1. BLOOM.
2. Epidermolisis de KOEBNER.
3. Enfermedad de DOWLING-MEARA.
4. Sndrome de COCKAYNE.
5. Sndrome de ROTHMUND.

214. Ante un RN que presenta gran dificultad respira-
toria y boca abierta sospecharemos que la causa
pueda ser:

1. Fibrosis pulmonar.
2. Atresia de coanas.
3. Parlisis diafragmtica.
4. Aspiracin meconial.
5. Enf. de la mb hialina.

215. La mas frecuente de las formas histologicas del
cancer de cuello uterino es:

1. Adenocarcinoma cervical
2. Carcinoma epidermoide
3. Carcinoma mixto
4. Sarcoma cervical
5. Carcinoma metastasico

216. La neoplasia mas frecuente de la mujer es:

1. Cancer de endometrio
2. Cancer de mama
3. Cancer de cervix uterino
4. Cancer de vulva
5. Cancer de ovario

217. El cancer de vulva se distribuye con mayor fre-
cuencia en el siguiente grupo de edad:

1. 40-49 aos
2. 50-59 aos
3. 60-69 aos
4. 70-79 aos
5. 80 o mas

218. La adenosis vaginal se caracteriza por:

1. Predispone siempre a adenocarcinoma.
2. Se manifiesta tempranamente en la infancia.
3. Est relacionada con el antecedente de ingesta
hormonal por parte de la madre de la mujer afecta.
4. La citologa es siempre sospechosa o maligna.
5. Se relaciona con la ingesta de testosterona.

219. La esofagitis por reflujo crnico puede conducir a
todas las siguientes complicaciones, excepto:

1. Hemorragia digestiva.
2. Estrechez esofgica pptica.
3. Anillo esofgico inferior.
4. Esfago de Barret.
5. Adenocarcinoma.

220. Una lcera gstrica puede llamarse benigna con
seguridad si:

1. Hay cicatrizacin significativa despus de un ciclo
teraputico de seis semanas con anticidos.
2. Tiene aspecto benigno en el estudio radiolgico
gastroduodenal con doble contraste aire-bario.
3. Se localiza en la curvatura menor gstrica.
4. Tiene menos de 2 cm.de dimetro.
5. No muestra signos de malignidad despus de 6
biopsias y estudios citolgicos del cepillado de la
mucosa.

221. En cul de las siguientes entidades la disfagia no
es de origen motor?

1. Acalasia.
2. Espasmo difuso de esfago.
3. Miastenia gravis.
4. S.de Plummer-Vinson.
5. Distrofia miotnica.

222. Paciente con clnica de dolor abdominal, prdida
de peso y hemorragia digestiva alta. La endosco-
pia muestra hiperplasia de pliegues gstricos. La
biopsia convencional no detecta alteraciones, slo
una hiperplasia epitelial sin atipias. Qu afirma-
cin es la correcta?

1. De tratarse de una sfilis, el diagnstico se confir-
mara por cultivo en medios celulares.
2. Se excluye el diagnstico de Enf.de Menetrier,
pues en sta nunca hay prdida significativa de pe-
so.
3. El primer diagnstico de sospecha es el adenocar-
cinoma gstrico.
4. El siguiente procedimiento diagnstico consistir
en la realizacin de una TAC para descartar un
gastrinoma.
5. El linfoma gstrico suele tener localizacin sub-
mucosa, por lo que su dgto.slo se descartar si
una biopsia profunda tampoco muestra clulas lin-
foides atpicas.

223. Mujer de 78 aos de edad con AP de procesos
bronconeumnicos de repeticin y diagnosticada
previamente de reflujo gastroesofgico en tto. con
medidas conservadoras.Acude a la consulta por
presentar disfagia orofarngea con sensacin de l-
quido en la nasofaringe y crisis de tos y asfixia con
la deglucin. En qu tipo de proceso patolgico
deberemos pensar ante este cuadro clnico?

1. Anillo de Schatzki.
2. Acalasia esofgica.
3. Divertculo de Zenker.
4. Estenosis esofgica pptica.
@
u
I
a

N
e
p
t
u
n
o
Centro de Estudios Aula Neptuno - GRANADA Ver.: 9/2001
www.aulaneptuno.com
- 21 -
5. Adenocarcinoma de esfago.

224. En el estudio de un sndrome malabsortivo se
obtiene una biopsia con infiltracin de eosinfiilos
de la mucosa y capa muscular sin signos de vascu-
litis. En este paciente no es esperable encontrar:

1. Eosinofilia perifrica.
2. Anemia ferropnica.
3. Dermatitis herpetiforme.
4. Esteatorrea.
5. Estenosis pilrica.

225. En relacin con la anatoma patolgica de la EII,
es falso que:

1. La presencia de abscesos crpticos es caracterstico
de colitis ulcerosa.
2. En un 15% de los casos de Enf.de Crohn, el recto
est respetado.
3. Lo ms caracterstico de la enf.de Crohn es la in-
flamacin crnica que afecta a todas las capas de
la pared intestinal.
4. La infeccin por yersinia enterocoltica puede
producir una iletis aguda inflamatoria autolimita-
da.
5. La inflamacin en la colitis ulcerosa es uniforme y
continua, sin reas intermedias de mucosa normal.

226. En un paciente con sospecha de enfermedad celia-
ca que no ha respondido al tratamiento diettico,
una nueva biopsia muestra una infiltracin masiva
de la lmina propia por clulas linfoides pleomr-
ficas con infiltracin y destruccin de las criptas. El
diagnstico ms probable es:

1. Espre colgeno.
2. Espre tropical.
3. Enfermedad de Whipple.
4. Linfangiectasia intestinal.
5. Linfoma intestinal.

227. Cul de las siguientes sera la siguiente prueba
ms indicada en un paciente en que se ha demos-
trado la presencia de esteatorrea?

1. Prueba del aliento con triolena marcada.
2. Biopsia intestinal.
3. Test de la D-xilosa oral.
4. Prueba del aliento con colilglicina marcada.
5. Prueba de la absorcin con bentiromida-PABA.

228. Nio de 5 aos, que padece dolor abdominal, aste-
nia, adelgazamiento y diarrea con heces pastosas y
brillantes, que flotan en el agua del inodoro. Se
aprecia asmismo retraso en el crecimiento, aftas
orales y lesiones vesiculo-ampollosas en codos y
rodillas, al parecer muy pruriginosas. El trnsito
intestinal muestra una imagen en nevada, con
edema de los pliegues en duodeno y primeras asas
yeyunales. La prueba del aliento con triolena
marcada confirma la presencia de malabsorcin.
Hay alteracin tambin de las pruebas de la D-
xilosa y de la tolerancia a la lactosa. Qu afirma-
cin es cierta en este nio?

1. El hallazgo en la biopsia de fibras colgenas en la
lmina propia significara un muy mal pronstico.
2. La nica medida diettica necesaria es suprimir el
gluten de la dieta.
3. En la biopsia encontraremos una atrofia de las
criptas, con hiperplasia vellositaria.
4. Est indicado iniciar tratamiento con vitamina D,
calcio, cido folnico y vitamina B12.
5. La enfermedad celiaca asociada a dermatitis her-
petiforme tiene un pronstico muy malo.

229. La asociacin de malabsorcin, hiperpigmenta-
cin, linfadenopatas y manifestaciones neurolgi-
cas, con artritis y fiebre, es muy sugerente de:

1. Linfangiectasia.
2. Enf.de Bassen Kornzweig.
3. Enf.de Whipple.
4. Enf.celiaca.
5. Espre no tropical.

230. Indica la respuesta falsa sobre las determinaciones
analticas en los pacientes que padecen pancreatitis
aguda:

1. En los casos en que la pancreatitis cursa con hiper-
lipemia los niveles de amilasa y lipasa pueden
mantenerse normales por interferencia de los lpi-
dos en la determinacin enzomtica.
2. La TC es la tcnica actualmente de eleccin para
la evaluacin de la gravedad de la pancreatitis
aguda, considerndose como signos de mayor
gravedad la presencia de colecciones lquidas y la
existencia de gas en el pncreas o en la zona adya-
cente.
3. La lipasa no slo se produce en el pncreas sino
tambin en intestino, rin y bazo, por lo que pro-
cesos inflamatorios en estos rganos tambin pue-
de cursar con cifras altas de este enzima.
4. La amilasa tiene una vida media ms larga (y por
tanto su elevacin es ms duradera) en los pacien-
tes con insuficiencia renal.
5. .La determinacin de tripsina y elastasa por ra-
dioinmunoanlisis son las tcnicas de eleccin pa-
ra el diagnstico de pancreatitis aguda en el servi-
cio de urgencias, dada su alta especificidad.

231. De entre los siguientes ttos. seala el de eleccin al
iniciar el tto. de un paciente cirrtico con ascitis le-
ve:

1. Anastomosis peritoneovenosa.
2. Dieta hiposdica y Furosemida.
3. Dieta hiposdica y Espironolactona.
4. Dieta hiposdica y Hidroclorotiazida.
5. Paracentesis evacuadora seguida de expansin de
volumen plasmtico con seroalbmina.

232. Cul de las siguientes entidades NO se asocia al
desarrollo de una insuf. heptica aguda grave? :

1. Enf. de Wilson.
2. Estaeasosis aguda masiva del embarazo.
3. Hemocromatosis idioptica.
4. TBC.
5. Hepatitis isqumica.

233. La seroconversin HBe slo puede indicar uno de
los siguientes datos:

1. La replicacin viral contina.
2. Sigue existiendo infeccin.
3. El paciente sigue siendo contagioso.
4. Un 10% evolucionar a la cronicidad.
5. Mal pronstico.

@
u
I
a

N
e
p
t
u
n
o
Centro de Estudios Aula Neptuno - GRANADA Ver.: 9/2001
www.aulaneptuno.com
- 22 -
234. Si un paciente asintomtico presenta HBsAg + e
IgG anti HBc +, podemos pensar que tiene:

1. Es un portador sano.
2. Infeccin persistente.
3. Infeccin aguda en fase de incubacin.
4. Ha sido vacunado.
5. Hepatitis crnica.

235. En la clasificacin de Child para la evaluacin
clnica del grado de insuficiencia heptica, cul de
los siguientes parmetros no se considera?

1. Bilirrubina srica.
2. Valores de presin portal.
3. Grado de encefalopata heptica.
4. Albmina srica.
5. Tasa de protrombina.

236. Cul de los siguientes frmacos utilizados en el
tratamiento de la lcera duodenal no debe indicar-
se en la lcera gstrica?

1. Anticidos.
2. Misoprostol.
3. Ranitidina.
4. Sucralfato.
5. Pirencepina.

237. Cul es la diferencia principal entre la Fibrosis
Heptica Congnita y la Enfermedad de Caroli?

1. La presencia de fibrosis en las vas biliares intra-
hepticas (FHC) o extrahepticas (Caroli).
2. La presencia de fibrosis en las vas biliares intra-
hepticas menores (FHC) o dilatacin de las vas
intrahepticas mayores (Caroli).
3. La etiologa congnita de la FHC y adquirida del
Caroli.
4. La asociacin con otras malformaciones de la
FHC y no del Caroli.
5. La FHC es un cuadro con afectacin del rbol bi-
liar mientras el Caroli afecta sobre todo al lecho
portal.

238. En un recin nacido con elevacin persistente de la
billirrubina directa, cul de estos hallazgos me
hace pensar en una atresia biliar extraheptica?

1. Megacoldoco en la radiografa simple.
2. Presencia de aire en el territorio portal en la radio-
grafa simple.
3. Ausencia de esplenomegalia.
4. Reduccin de la excrecin fecal de Rosa de Ben-
gala.
5. Retinitis pigmentaria.

239. La obstruccin adquirida de la va lacrimal por
debajo del saco lacrimal (en el conducto lacrimo-
nasal), se corrige con una operacin de:

1. Extirpacin de glndula lacrimal.
2. Dacriocistorrinostoma.
3. Sondeo de va lacrimal.
4. Intubacin de va lacrimal.
5. Acortamiento palpebral.

240. Cul de las siguientes alteraciones oculares est
producida por la osteognesis imperfecta?

1. Catarata congnita.
2. Desprendimiento de retina.
3. Esclerticas azuladas.
4. Atrofia del nervio ptico.
5. Iritis.

241. En cul de estas afecciones no hay papiledema?

1. Hipertensin arterial.
2. Papilitis.
3. Glaucoma crnico.
4. Leucemia.
5. Hipertensin intracraneal.

242. Qu frmaco de los que se emplean de forma
crnica en el tto. del glaucoma se ha relacionado
con la aparicin de Sncopes por BAV completo? :

1. Fenilefrina.
2. Pilocarpina.
3. Acetazolamida.
4. Timolol.
5. Carbacol.

243. NO se ha relacionado como causa de Glaucoma
secundario a una de las siguientes :

1. Sd. de Sturge-Weber.
2. Hifema.
3. Catarata hipermadura.
4. Trombosis de la vena central de la retina.
5. Retinoblastoma.

244. Un varn de 25 aos de aspecto sano busca aseso-
ra sobre el riesgo de tener un nio con Trisoma
21, porque su hermana de 22 aos acaba de tener
un nio con el trastorno. Su mdico le asegura que
el riesgo es insignificante porque no hay otro ante-
cedente familiar de Trisoma 21. Un ao despus se
le informa que tuvo un nio con Trisoma 21. La
explicacin ms factible es :

1. La esposa del paciente, de 26 aos, produjo la no
disyuncin cromosmica que dio por resultado el
nio trismico.
2. El paciente y su hermana tenan una translocacin
equilibrada.
3. Tanto la hermana como el paciente eran trismi-
cos pero sin expresin de su estado de trisoma.
4. Haba un matrimonio consanguneo.
5. Su mdico es portador de la trisoma 21.

245. La tcnica donde se busca una se-cuencia especfi-
ca de ARN con hi-bridacin de una sonda en
fragmen-tos de ARN, primero separados en un gel
y luego transferidos a un so-porte inerte como ny-
lon, recibe el nombre de:

1. Western bloting.
2. Northern bloting.
3. Southern bloting.
4. Hibridacin "In situ".
5. Reaccin de PCR.

246. Si estudio el nmero de molculas de sodio que,
independientemente y al azar, atraviesan una
membrana semipermeable, la variable seguir
una distribucin:

1. Cuantitativa continua.
2. Binomial.
3. Poisson.
@
u
I
a

N
e
p
t
u
n
o
Centro de Estudios Aula Neptuno - GRANADA Ver.: 9/2001
www.aulaneptuno.com
- 23 -
4. Normal.
5. Normal tipificada.

247. La distribucin de los distintos tipos (I a V) de
hiperlipoproteinemias es una variable:

1. Cuantitativa discreta.
2. Cualitativa simple.
3. Cualitativa ordinal.
4. Cuantitativa continua.
5. Cualitativa dicotmica.

248. Respecto a las pruebas de significacin, seala lo
falso:

1. Es un procedimiento por el cual nos decidimos
por la hiptesis nula (Ho) o la alternativa (H1).
2. La Ho es la hiptesis de la "no diferencia".
3. Al realizar una prueba estadstica de significacin,
determinamos la probabilidad de que H1 sea cier-
ta.
4. La H1 es la que se aceptar si el resultado de la
prueba permite rechazar la Ho.
5. La significacin estadstica es la condicin resul-
tante del rechazo de la Ho mediante las pruebas de
significacin.

249. Medimos la altura de los nios de quinto curso de
EGB (o como se llame ESO) del colegio Dulce
Nombre de Mara (PP.Escolapios). La media es de
150 centmetros, la desviacin tpica, 12 centme-
tros y el error estndar, 2 centmetros. Qu afir-
macin es correcta?

1. El 95% de los nios del colegio mide
entre 126 y 174 centmetros.
2. Existe un 95% de confianza de que la
verdadera altura media de los nios del colegio
(desde los ms pitusines hasta COU) se site entre
146 y 154 centmetros.
3. El 95% de los nios medidos tienen
una altura comprendida entre 126 y 174 centme-
tros.
4. El 40% de los nios del colegio son la
mitad.
5. El 5% de los nios del curso podran
entrar en la "Operacin altura" de la Federacin
Espaola de Baloncesto, por medir ms de 174
centmetros.

250. La supervivencia media en una poblacin de pa-
cientes con cncer est establecida en 38,3 meses y
se quiere estudiar el efecto de un nuevo frmaco en
el tiempo de supervivencia. Qu prueba estadsti-
ca debe utilizarse?

1. Prueba de los rangos con signo de Wilcoxon.
2. Prueba de los Timberwolves de Minnessota al
perfume de albahaca.
3. T de Student para una muestra.
4. Eau d't (con perdn).
5. U de Mann Whitney.

251. En relacin al proceder diagnstico en los pacien-
tes con hidrocefalia normotensiva, es cierto que:

1. La triada clnica de demencia, alteracin de la
marcha e incontinencia urinaria en un varn de
ms de 50 aos es diagnstico de certeza de hidro-
cefalia normotensiva, por lo que debe iniciarse tra-
tamiento sin realizar ms pruebas.
2. La TAC es diagnstica al mostrar dilatacin de los
ventrculos y atrofia de las circunvoluciones.
3. La monitorizacin continua de la presin del LCR
durante la noche puede revelar elevaciones an-
malas intermitentes de la presin, diagnsticas de
un trastorno en la absorcin del LCR.
4. En todo paciente con sospecha de hidrocefalia
normotensiva es imprescindible la realizacin de
cisternografa isotpica para confirmar el diagns-
tico.
5. La resonancia magntica no tiene utilidad en el
protocolo diagnstico de la HNT.

252. Seala el tumor heptico ms comn en la edad
adulta :

1. Ca hepatocelular.
2. Hemoangioma.
3. Hepatoblastoma.
4. Metstasico.
5. Hiperplasia nodular focal.

253. Un jubilado de 81 aos sufre un fracaso rpido de
la funcin renal tras la realizacin de una corona-
riografa. En el sedimento urinario hay proteinu-
ria, hematuria y eosinofiluria. Cul de las siguien-
tes pruebas complementarias puede ayudarte ms
a establecer el diagnstico?

1. Determinacin de los niveles sricos de IgE.
2. Arteriografa renal.
3. Examen del fondo de ojo.
4. Determinacin de la actividad renina plasmtica.
5. Perfil de LDH y transaminasas en las siguientes
48 horas.

254. Un paciente que presenta eritema facial, cervical y
torcico de inicio relativamente brusco, aprecin-
dose en el dorso de los dedos ppulas hiperquera-
tsicas junto a hiperqueratosis palmoplantar ama-
rillenta, es muy probable que padezca:

1. Pitiriasis rubra pilaris.
2. Dermatitis seborreica.
3. Pitiriasis rosada de Gibert.
4. Enfermedad de Darier.
5. Urticaria pigmentada.

255. El sndrome hereditario provocado por una alte-
racin en el cromosoma 2 y caracterizado por una
incapacidad de los melanocitos para emigrar des-
de la cresta neural con sordera y alteraciones pig-
mentarias y cutneas se denomina:

1. Sndrome de Alport.
2. Neurofibromatosis tipo 2.
3. Sndrome de Alport.
4. Sndrome de Waardenburg.
5. Sndrome de Refsum.

256. Nio de ocho aos con fiebre, catarro, manchas
rojas petequiales en el velo del paladar y adenopa-
tas retroauriculares, cervicales y suboccipitales.
Posteriormente:exantema maculopapuloso. Dgto:

1. Rubeola.
2. Sarampin.
3. Exantema sbito.
4. Kawasaki.
5. Megaleritema.

@
u
I
a

N
e
p
t
u
n
o
Centro de Estudios Aula Neptuno - GRANADA Ver.: 9/2001
www.aulaneptuno.com
- 24 -
257. Paciente con A. reumatoidea de larga evolucin
est siendo estudiado por proceso anmico crnico.
Entre los siguientes datos hay uno que no coincide
con el diagnstico de Anemia de las enfermedades
crnicas:

1. VCM = 89 fl.
2. Indice de saturacin de transferrina > 15%.
3. Descenso de ferritina srica.
4. Descenso de transferrina srica.
5. Ferropenia.

258. Varon de 50 aos con episodios repetidos de is-
quemia cerebral transitoria consistente en pridad
de fuerza y paresias en brazo y pierna derecha y
amaurosis fugaz de ojo izquierdo. Presenta esteno-
sis del 75% en inicio de carotida interna izquierda.
Cul es la actitud correcta?:

1. Anticoagulacion con dicumarnicos 6-12 meses
2. Anticoagulacion con Heparina 1 semana
3. Anticoagulacion con Heparina y antiagregantes
plaquetarios
4. Endarterectomia de cartida interna izquierda
5. By-pass aorto-carotideo con vena safena autloga

259. Cul de las siguientes determinaciones permite
valorar mejor el grado de afectacion fetal intrate-
ro en un caso de incompatibilidad Rh?:

1. Concentracion de anticuerpos anti-Rh en compar-
timento materno
2. Niveles de bilirrubina en compartimento materno
3. Niveles de bilirrubina en lquido amniotico
4. Concentracion de anticuerpos anti-Rh en liquido
amniotico
5. Medicion ecogrfica del diametro biparietal del fe-
to

260. El tratamiento ms habitual de las lesiones agudas
de la mucosa gstrica consiste en:

1. Omeprazol y Bismuto.
2. Claritromicina y omeprazol.
3. Anticidos y ranitidina.
4. Sucralfato y pirencepina.
5. Bismuto y carbenoxolona.

S-ar putea să vă placă și